PedsPart#@

¡Supera tus tareas y exámenes ahora con Quizwiz!

Four-year-old David is placed in Buck extension traction for Legg-Calvé-Perthes disease. He is crying with pain as the nurse assesses that the skin of his right foot is pale with an absence of pulse. Which action should the nurse take first? a. Notify the practitioner of the changes noted. b. Give the child medication to relieve the pain. c. Reposition the child and notify physician. d. Chart the observations and check the extremity again in 15 minutes.

ANS: A The absence of a pulse and change in color of the foot must be reported immediately for evaluation by the practitioner. Pain medication should be given after the practitioner is notified. Legg-Calvé-Perthes disease is an emergency condition; immediate reporting is indicated. The findings should be documented with ongoing assessment.

46. Which is an important nursing consideration in the care of a child with celiac disease? a. Refer to a nutritionist for detailed dietary instructions and education. b. Help child and family understand that diet restrictions are usually only temporary. c. Teach proper hand washing and standard precautions to prevent disease transmission. d. Suggest ways to cope more effectively with stress to minimize symptoms.

ANS: A The main consideration is helping the child adhere to dietary management. Considerable time is spent explaining to the child and parents about the disease process, the specific role of gluten in aggravating the condition, and foods that must be restricted. Referral to a nutritionist would help in this process. The most severe symptoms usually occur in early childhood and adult life. Dietary avoidance of gluten should be lifelong. Celiac disease is not transmissible or stress related.

The nurse is taking care of an adolescent with osteosarcoma. The parents ask the nurse about treatment. The nurse should make which accurate response about treatment for osteosarcoma? a. Treatment usually consists of surgery and chemotherapy. b. Amputation of affected extremity is rarely necessary. c. Intensive irradiation is the primary treatment. d. Bone marrow transplantation offers the best chance of long-term survival.

ANS: A The optimal therapy for osteosarcoma is a combination of surgery and chemotherapy. Intensive irradiation and bone marrow transplantation are usually not part of the therapeutic management.

The nurse should monitor for which effect on the cardiovascular system when a child is immobilized? a. Venous stasis b. Increased vasopressor mechanism c. Normal distribution of blood volume d. Increased efficiency of orthostatic neurovascular reflexes

ANS: A The physiologic effects of immobilization, as a result of decreased muscle contraction, include venous stasis. This can lead to pulmonary emboli or thrombi. A decreased vasopressor mechanism results in orthostatic hypotension, syncope, hypotension, decreased cerebral blood flow, and tachycardia. An altered distribution of blood volume is found with decreased cardiac workload and exercise tolerance. Immobilization causes a decreased efficiency of orthostatic neurovascular reflexes with an inability to adapt readily to the upright position and with pooling of blood in the extremities in the upright position.

The nurse is caring for an infant with developmental dysplasia of the hip. Which clinical manifestations should the nurse expect to observe? (Select all that apply.) a. Positive Ortolani click b. Unequal gluteal folds c. Negative Babinski sign d. Trendelenburg sign e. Telescoping of the affected limb f. Lordosis

ANS: A, B A positive Ortolani test and unequal gluteal folds are clinical manifestations of developmental dysplasia of the hip seen from birth to 2 to 3 months. Unequal gluteal folds, negative Babinski sign, and Trendelenburg sign are signs that appear in older infants and children. Telescoping of the affected limb and lordosis are not clinical manifestations of developmental dysplasia of the hip.

An adolescent with juvenile idiopathic arthritis (JIA) is prescribed abatacept (Orencia). Which should the nurse teach the adolescent regarding this medication? (Select all that apply.) a. Avoid receiving live immunizations while taking the medication. b. Before beginning this medication, a tuberculin screening test will be done. c. You will be getting a twice-a-day dose of this medication. d. This medication is taken orally.

ANS: A, B Abatacept reduces inflammation by inhibiting T cells and is given intravenously every 4 weeks. Possible side effects of biologics include an increased infection risk. Because of the infection risk, children should be evaluated for tuberculosis exposure before starting these medications. Live vaccines should be avoided while taking these agents.

24. The nurse is caring for a child admitted with acute abdominal pain and possible appendicitis. Which is appropriate to relieve the abdominal discomfort? a. Place in Trendelenburg position. b. Allow to assume position of comfort. c. Apply moist heat to the abdomen. d. Administer a saline enema to cleanse bowel.

ANS: B The child should be allowed to take a position of comfort, usually with the legs flexed. The Trendelenburg position will not help with the discomfort. In any instance in which appendicitis is a possibility, there is a danger in administering a laxative or enemas or applying heat to the area. Such measures stimulate bowel motility and increase the risk of perforation.

37. During the first few days after surgery for cleft lip, which intervention should the nurse do? a. Leave infant in crib at all times to prevent suture strain. b. Keep infant heavily sedated to prevent suture strain. c. Remove restraints periodically to cuddle infant. d. Alternate position from prone to side-lying to supine.

ANS: C Remove restraints periodically, while supervising the infant, to allow him or her to exercise arms and to provide cuddling and tactile stimulation. The infant should not be left in the crib, but should be removed for appropriate holding and stimulation. Analgesia and sedation are administered for pain. Heavy sedation is not indicated. The child should not be placed in the prone position.

7. Which pathogen is the viral pathogen that frequently causes acute diarrhea in young children? a. Giardia organisms b. Shigella organisms c. Rotavirus d. Salmonella organisms

ANS: C Rotavirus is the most frequent viral pathogen that causes diarrhea in young children. Giardia (parasite) and Salmonella are bacterial pathogens that cause diarrhea. Shigella is a bacterial pathogen that is uncommon in the United States.

Which type of traction uses skin traction on the lower leg and a padded sling under the knee? a. Dunlop b. Bryant c. Russell d. Buck extension

ANS: C Russell traction uses skin traction on the lower leg and a padded sling under the knee. The combination of longitudinal and perpendicular traction allows realignment of the lower extremity and immobilizes the hips and knees in a flexed position. Dunlop traction is an upper-extremity traction used for fractures of the humerus. Bryant traction is skin traction with the legs flexed at a 90-degree angle at the hip. Buck extension traction is a type of skin traction with the legs in an extended position. It is used primarily for short-term immobilization, preoperatively with dislocated hips, for correcting contractures, or for bone deformities such as Legg-Calvé-Perthes disease.

23. When caring for a child with probable appendicitis, the nurse should be alert to recognize that which condition or symptom is a sign of perforation? a. Bradycardia b. Anorexia c. Sudden relief from pain d. Decreased abdominal distention

ANS: C Signs of peritonitis, in addition to fever, include sudden relief from pain after perforation. Tachycardia, not bradycardia, is a manifestation of peritonitis. Anorexia is already a clinical manifestation of appendicitis. Abdominal distention usually increases.

18. A 3-year-old child with Hirschsprung disease is hospitalized for surgery. A temporary colostomy will be necessary. The nurse should recognize that preparing this child psychologically is: a. not necessary because of child's age. b. not necessary because colostomy is temporary. c. necessary because it will be an adjustment. d. necessary because the child must deal with a negative body image.

ANS: C The child's age dictates the type and extent of psychological preparation. When a colostomy is performed, the child who is at least preschool age is told about the procedure and what to expect in concrete terms, with the use of visual aids. It is necessary to prepare a 3-year-old child for procedures. The preschooler is not yet concerned with body image.

41. Which observation made of the exposed abdomen is most indicative of pyloric stenosis? a. Abdominal rigidity b. Substernal retraction c. Palpable olive-like mass d. Marked distention of lower abdomen

ANS: C The diagnosis of pyloric stenosis is often made after the history and physical examination. The olive-like mass is easily palpated when the stomach is empty, the infant is quiet, and the abdominal muscles are relaxed. Abdominal rigidity and substernal retraction are usually not present. The upper abdomen, not lower abdomen, is distended.

The nurse will soon receive a 4-month-old who has been diagnosed with intussusception. The infant is described as very lethargic with the following vital signs: T 101.8°F (38.7°C), HR 181, BP 68/38. The reporting nurse states the infant's abdomen is very rigid. Which is the most appropriate action for the receiving nurse? 1. Prepare to accompany the infant to a computed tomography scan to confirm the diagnosis. 2. Prepare to accompany the infant to the radiology department for a reducing enema. 3. Prepare to start a second intravenous line to administer fluids and antibiotics. 4. Prepare to get the infant ready for immediate surgical correction.

4. Prepare to get the infant ready for immediate surgical correction. Intussusception with peritonitis is a surgical emergency, so preparing the infant for surgery is the nurse's top priority.

1. Which condition in a child should alert a nurse for increased fluid requirements? a. Fever b. Mechanical ventilation c. Congestive heart failure d. Increased intracranial pressure (ICP)

ANS: A Fever leads to great insensible fluid loss in young children because of increased body surface area relative to fluid volume. Respiratory rate influences insensible fluid loss and should be monitored in the mechanically ventilated child. Congestive heart failure is a case of fluid overload in children. Increased ICP does not lead to increased fluid requirements in children.

A nurse working at a children's hospital receives report on four patients who were just admitted to the unit within the past hour. Which nurse should the nurse assess first? 1) A 9-month-old infant who has been vomiting for the past 12 hours who has a fever of 100.3. 2) A 4-month-old who is resting quietly right now with reports of jelly-like stools and severe pain for the past 6 hours. 3) A 2-year-old who with a fractured femur who was medicated 30 minutes ago for pain. 4) A 3-month-old who has passed runny stools frequently overnight with sunken anterior and posterior fontanelles.

Answer: 2) Jelly-like stools and severe pain indicate intussusception, and this is a medical emergency. Afterwards, the nurse should assess the child with sunken fontanelles, as this indicates severe dehydration.

The nurse is caring for a school-age child diagnosed with juvenile idiopathic arthritis (JIA). Which intervention should be a priority? a. Apply ice packs to relieve stiffness and pain. b. Administer acetaminophen to reduce inflammation. c. Teach the child and family correct administration of medications. d. Encourage range-of-motion exercises during periods of inflammation.

ANS: C The management of JIA is primarily pharmacologic. The family should be instructed regarding administration of medications and the value of regular schedule of administration to maintain a satisfactory blood level in the body. They need to know that NSAIDs should not be given on an empty stomach and to be alert for signs of toxicity. Warm moist heat is best for relieving stiffness and pain. Acetaminophen does not have antiinflammatory effects. Range-of-motion exercises should not be done during periods of inflammation.

Which medication is usually tried first when a child is diagnosed with juvenile idiopathic arthritis (JIA)? a. Aspirin b. Corticosteroids c. Cytotoxic drugs such as methotrexate d. Nonsteroidal anti-inflammatory drugs (NSAIDs)

ANS: D NSAIDs are the first drugs used in JIA. Naproxen, ibuprofen, and tolmetin are approved for use in children. Aspirin, once the drug of choice, has been replaced by the NSAIDs because they have fewer side effects and easier administration schedules. Corticosteroids are used for life-threatening complications, incapacitating arthritis, and uveitis. Methotrexate is a second-line therapy for JIA.

9. A child is admitted with bacterial gastroenteritis. Which lab results of a stool specimen confirm this diagnosis? a. Eosinophils b. Occult blood c. pH less than 6 d. Neutrophils and red blood cells

ANS: D Neutrophils and red blood cells in stool indicate bacterial gastroenteritis. Protein intolerance and parasitic infections are suspected in the presence of eosinophils. Occult blood may indicate pathogens such as Shigella, Campylobacter, or hemorrhagic Escherichia coli strains. A pH of less than 6 may indicate carbohydrate malabsorption or secondary lactase insufficiency.

22. Which clinical manifestation would be the most suggestive of acute appendicitis? a. Rebound tenderness b. Bright red or dark red rectal bleeding c. Abdominal pain that is relieved by eating d. Abdominal pain that is most intense at McBurney point

ANS: D Pain is the cardinal feature. It is initially generalized, usually periumbilical. The pain localizes to the right lower quadrant at McBurney point. Rebound tenderness is not a reliable sign and is extremely painful to the child. Bright red or dark red rectal bleeding and abdominal pain that is relieved by eating are not signs of acute appendicitis.

The nurse is conducting teaching to parents of a 7-year-old child who fractured an arm and is being discharged with a cast. Which instruction should be included in the teaching? a. Swelling of the fingers is to be expected for the next 48 hours. b. Immobilize the shoulder to decrease pain in the arm. c. Allow the affected limb to hang down for 1 hour each day. d. Elevate casted arm when resting and when sitting up.

ANS: D The injured extremity should be kept elevated while resting and in a sling when upright. This will increase venous return. Swelling of the fingers may indicate neurovascular damage and should be reported immediately. Permanent damage can occur within 6 to 8 hours. Joints above and below the cast on the affected extremity should be moved. The affected limb should not hang down for any length of time.

34. Caring for the newborn with a cleft lip and palate before surgical repair includes: a. gastrostomy feedings. b. keeping infant in near-horizontal position during feedings. c. allowing little or no sucking. d. providing satisfaction of sucking needs.

ANS: D Using special or modified nipples for feeding techniques helps meet the infant's sucking needs. Gastrostomy feedings are usually not indicated. Feeding is best accomplished with the infant's head in an upright position. The child requires both nutritive and nonnutritive sucking.

A 2-year-old child with acute diarrhea has been diagnosed with mild dehydration. Which rehydration methods would the nurse expect the health care provider to prescribe? 1. Increase intake of water with a diet high in carbohydrates. 2. Consume oral rehydration fluid, advancing to a regular diet. 3. Begin fluid replacement immediately with intravenous fluids. 4. Begin a diet of bananas, rice, apples, pears, and toast with juice.

Mild dehydration is usually treated at home and consists of age-appropriate diet along with oral rehydration fluids. Bananas, rice, apples, pears, and toast with juice can be irritating to the gastrointestinal (GI) tract and does not provide the rehydration needed in a child who is dehydrated. Water does not provide electrolyte fluid replacement, a need during dehydration. Hospitalization and intravenous fluids is not required with mild dehydration.

A group of student nurses are reviewing nursing diagnoses for cleft lip and cleft palate. The students recognize which of the following as priority nursing diagnosis for children with cleft lip and cleft palate? 1) Ineffective bonding related to inability to maintain effective mother-child feeding habits 2) Altered nutrition: less than body requirements related to excessive feeding time and child fatigue 3) Risk for altered self perception related to nasal quality of speech and delays in speech development 4) Risk for infection related to build-up of fluid in the middle ear and chronic otitis media

Answer: 2) Our biggest concerns in a child with cleft lip and cleft palate are nutrition (including the risk for aspiration/choking) and bonding. However, nutrition trumps bonding.

36. The nurse is providing care to a child with a long-leg hip spica cast. What is the priority nursing diagnosis? A. Risk for impaired skin integrity due to cast and location B. Deficient knowledge related to cast care C. Risk for delayed development related to immobility D. Self-care deficit related to immobility

Answer: A Rationale: Although deficient knowledge, risk for delayed development, and self-care deficit may be applicable, the child is at increased risk for skin breakdown due to the size of the cast and its location. In addition, the cast has an opening, which allows for elimination. Soiling of cast edges or leakage of urine or stool can lead to skin breakdown.

26. The nurse is caring for an infant with osteogenesis imperfecta and is providing instruction on how to reduce the risk of injury. Which response from the mother indicates a need for further teaching? A. "I need to avoid pushing or pulling on an arm or leg." B. "I must carefully lift the baby from under the armpits." C. "I should not bend an arm or leg into an awkward position." D. "We must avoid lifting the legs by the ankles to change diapers."

Answer: B Rationale: The nurse needs to emphasize that the mother must not lift a baby or young child with osteogenesis imperfecta from under the armpits as it may cause harm. Avoiding pushing or pulling, not bending an arm or leg into an awkward position, and avoiding lifting the legs by the ankles are appropriate responses.

The parents of a newborn with a cleft lip are concerned and ask the nurse when the lip will be repaired. With which statement should the nurse respond? 1. Cleft lip cannot be repaired. 2. Cleft-lip repair is usually performed by 6 months of age. 3. Cleft-lip repair is usually performed during the first weeks of life. 4. Cleft-lip repair is usually performed between 6 months and 2 years.

3. Cleft-lip repair is usually performed during the first few weeks of life. Early repair may improve bonding and makes feeding much easier. Revisions may be required at a later age. All other options are incorrect.

The nurse has provided dietary instructions to the mother of a child with celiac disease. The nurse determines that further instruction is needed if the mother states that she will include which food item in the child's nutritional plan? 1. Corn 2. Chicken 3. Oatmeal 4. Vitamin supplements

3. Dietary management is the mainstay of treatment for the child with celiac disease. All wheat, rye, barley, and oats should be eliminated from the diet and replaced with corn and rice. Vitamin supplements, especially fat-soluble vitamins and folate, may be needed in the early period of treatment to correct deficiencies.

7. The nurse is caring for a 9-month-old with diarrhea secondary to rotavirus. The child has not vomited and is mildly dehydrated. Which is likely to be included in the discharge teaching? 1. Administer Imodium as needed. 2. Administer Kaopectate as needed. 3. Continue breastfeeding per routine. 4. The infant may return to day care 24 hours after antibiotics have been started.

3. Continue breastfeeding per routine. Breastfeeding is usually well tolerated and helps prevent death of intestinal villi and malabsorption.

The nurse is caring for a 3-month-old being evaluated for possible Hirschsprung disease. His parents call the nurse and show her his diaper containing a large amount of mucus and bloody diarrhea. The nurse notes that the infant is irritable and his abdomen appears very distended. Which should be the nurse's next action? 1. Reassure the parents that this is an expected finding and not uncommon. 2. Call a code for a potential cardiac arrest, and stay with the infant. 3. Immediately obtain all vital signs with a quick head-to-toe assessment. 4. Obtain a stool sample for occult blood.

3. Immediately obtain all vital signs with a quick head-to-toe assessment. All vital signs need to be evaluated because the child with enterocolitis can quickly progress to a state of shock. A quick head-to-toe assessment will allow the nurse to evaluate the child's circulatory system.

Parents bring their child to the emergency department and tell the nurse that the child has been complaining of colicky abdominal pain located in the lower right quadrant of the abdomen. The nurse suspects that the child has which disorder? 1. Peritonitis 2. Appendicitis 3. Intussusception 4. Hirschsprung's disease

2. The most common symptom of appendicitis is a colicky, periumbilical, or lower abdominal pain located in the right quadrant. Peritonitis is a complication that can follow organ perforation or intestinal obstruction. The classic signs and symptoms of intussusception are acute, colicky abdominal pain with currant jelly-like stools. Clinical manifestations of Hirschsprung's disease include constipation, abdominal distension, and ribbon-like, foul-smelling stools.

The nurse provides home care instructions to the mother of a child who had a cleft palate repair 4 days ago. Which statement by the mother indicates the need for further instruction? 1. "I will use a short nipple on the bottle." 2. "I need to buy some straws for drinking." 3. "I can give my child the pacifier in 2 weeks." 4. "I may give my baby food mixed with water."

2. The mother needs to be instructed that straws, pacifiers, spoons, and fingers must be kept away from the child's mouth for 7 to 10 days. Additionally, the mother should be advised to avoid taking an oral temperature. The remaining options are accurate measures to implement after cleft palate repair.

The parents of a child being evaluated for appendicitis tell the nurse the physician said their child has a positive Rovsing sign. They ask the nurse what this means. Select the nurse's best response. 1. "Your child's physician should answer that question." 2. "A positive Rovsing sign means the child feels pain in the right side of the abdomen when the left side is palpated." 3. "A positive Rovsing sign means pain is felt when the physician removes the hand from the abdomen." 4. "A positive Rovsing sign means pain is felt in the right lower quadrant when the child coughs."

2. "A positive Rovsing sign means the child feels pain in the right side of the abdomen when the left side is palpated."

30. The nurse is preparing to administer intravenous fluids to manage a child with dehydration. The medical record indicates the child weighs 60 lb (27.2 kg). How many milliliters will initially be administered? Record your answer using two decimal places.

Ans: 545.45 Feedback: Nursing goals for the infant or child with dehydration are aimed at restoring fluid volume and preventing progression to hypovolemia. Provide oral rehydration to children for mild to moderate states of dehydration. Children with severe dehydration should receive intravenous fluids. Initially, administer 20 mL/kg of normal saline or lactated Ringer, and then reassess the hydration status.

1. The nurse is teaching the mother of a 5-year-old boy with a history of impaction how to administer enemas at home. Which response from the mother indicates a need for further teaching? A) "I should position him on his abdomen with knees bent." B) "He will require 250 to 500 mL of enema solution." C) "I should wash my hands and then wear gloves." D) "He should retain the solution for 5 to 10 minutes."

Ans: A Feedback: A 5-year-old child should lie on his left side with his right leg flexed toward the chest. An infant or toddler is positioned on his abdomen. Using 250 to 500 mL of solution, washing hands and wearing gloves, and retaining the solution for 5 to 10 minutes are appropriate responses.

7. A nurse is caring for a 14-year-old girl scheduled for a barium swallow/upper gastrointestinal (GI) series. Before providing instructions, what would be the priority? A) Screening the girl for pregnancy B) Reminding her to drink plenty of fluids after the procedure C) Ordering a bowel preparation D) Reminding the girl about potential light-colored stools

Ans: A Feedback: Females of reproductive age must be screened for pregnancy prior to the test because radiography is used. A bowel preparation is not necessary for a barium swallow/upper GI series. The reminders about fluids and light-colored stools are appropriate but are not the first priority.

After hydrostatic reduction for intussusception, the nurse should expect to observe which client response? 1. Abdominal distension 2. Currant jelly-like stools 3. Severe colicky-type pain with vomiting 4. Passage of barium or water-soluble contrast with stools

4. Intussusception is the telescoping of one portion of the bowel into another. Hydrostatic reduction may be necessary to resolve the condition. After hydrostatic reduction, the nurse observes for the passage of barium or water-soluble contrast material with stools. Abdominal distension and currant jelly-like stools are clinical indicators of intussusception. Colicky pain and vomiting are signs of an unresolved gastrointestinal disorder.

A 1-year-old child is diagnosed with intussusception, and the mother of the child asks the student nurse to describe the disorder. Which statement by the student nurse indicates correct understanding of this disorder? 1. "It is an acute bowel obstruction." 2. "It is a condition that causes an acute inflammatory process in the bowel." 3. "It is a condition in which a distal segment of the bowel prolapses into a proximal segment of the bowel." 4. "It is a condition in which a proximal segment of the bowel prolapses into a distal segment of the bowel."

4. Intussusception occurs when a proximal segment of the bowel prolapses into a distal segment of the bowel. It is not an acute bowel obstruction, but it is a common cause of bowel obstruction in infants and young children. It is not an inflammatory process.

26. A group of nursing students are reviewing information about inflammatory bowel disease in preparation for a class discussion on the topic. The students demonstrate understanding of the material when they identify which characteristics of Crohn disease? Select all that apply. A) Distributed in a continuous fashion B) Most common between the ages of 10 and 20 years C) Elevated erythrocyte sedimentation rate D) Low serum iron levels E) Tenesmus F) Loss of haustra within bowel

Ans: B, C, D Feedback: Crohn disease is most common between the ages of 10 and 20 years. Erythrocyte sedimentation rate is elevated, and serum iron levels are low. Ulcerative colitis is distributed continuously distal to proximal, with tenesmus and loss of haustra within the bowel. Crohn disease is segmental, with disease-free skip areas common, and the bowel wall has a cobblestone appearance.

14. The parent of a child diagnosed with osteomyelitis asks how the child acquired the illness. Which is the nurse's best response? 1. "Direct inoculation of the bone from stepping barefoot on a sharp stick." 2. "An infection from a scratched mosquito bite carried the infection through the bloodstream to the bone." 3. "The blood supply to the bone was disrupted because of the child's diabetes." 4. "An infection of the upper respiratory tract."

2. Infection through the bloodstream is the most likely cause of osteomyelitis in a child.

The nurse is writing out discharge instructions for the parents of a child diagnosed with celiac disease. The nurse should focus primarily on which aspect of care? 1. Restricting activity 2. Following a gluten-free diet 3. Following a lactose-free diet 4. Giving medication to manage the condition

2. The primary nursing consideration in the care of a child with celiac disease is to instruct the child and parents regarding proper dietary management. Although medications may be prescribed for the client with celiac disease, treatment focuses primarily on maintaining a gluten-free diet. The remaining options are not directly related to the care of a child with celiac disease.

The nurse is reviewing the plan of care for a child with a diagnosis of suspected appendicitis. The nurse would question which intervention if noted in the plan of care? 1. Taking the child's temperature with an oral thermometer 2. Applying a heating pad to abdomen to promote pain relief 3. Palpating between the right anterior superior iliac crest and umbilicus 4. Obtaining blood for complete blood count while starting an intravenous line

2. Whenever appendicitis is suspected, the nurse should be aware of the danger of administering laxatives or enemas or applying heat to the area. Such measures stimulate bowel motility and increase the risk of perforation. The nurse can determine the most intense site of pain, located at McBurney's point, by palpation. McBurney's point is midway between the right anterior superior iliac crest and the umbilicus. It is usually the location of greatest pain in the child with appendicitis. There is no contraindication to using an oral thermometer in a child with suspected appendicitis. Obtaining blood for a complete blood count is important to determine the white blood cell count.

The nurse is caring for an 8-week-old infant being evaluated for pyloric stenosis. Which statement by the parent would be typical for a child with this diagnosis? 1. "The baby is a very fussy eater and just does not want to eat." 2. "The baby tends to have a very forceful vomiting episode about 30 minutes after most feedings." 3. "The baby is always hungry after vomiting so I refeed." 4. "The baby is happy in spite of getting really upset after spitting up."

3. "The baby is always hungry after vomiting so I refeed." Infants with pyloric stenosis are always hungry and often appear malnourished.

1. Which would the nurse expect to assess on a 3-week-old infant with developmental dysplasia of the hip (DDH)? 1. Excessive hip abduction. 2. Femoral lengthening of an affected leg. 3. Asymmetry of gluteal and thigh folds. 4. Pain when lying prone.

3. In DDH, asymmetrical thigh and gluteal folds are frequently present.

During the initial assessment of a child admitted to the pediatric unit with osteomyelitis of the left tibia, when assessing the area over the tibia, which is an expected finding? 1. Diffuse tenderness. 2. Decreased pain. 3. Increased warmth. 4. Localized edema.

3. Increased warmth.

Which would the nurse expect to be included in the diagnostic workup of a child with suspected celiac disease? 1. Obtain complete blood count and serum electrolytes. 2. Obtain complete blood count and stool sample; keep child NPO. 3. Obtain stool sample and prepare child for jejunal biopsy. 4. Obtain complete blood count and serum electrolytes; monitor child's response to gluten-containing diet.

3. Obtain stool sample and prepare child for jejunal biopsy. A stool sample for analysis of fat and a jejunal biopsy can confirm the diagnosis.

A child is diagnosed with chronic constipation that has been unresponsive to dietary and activity changes. Which pharmacological measure is most appropriate? 1. Natural supplements and herbs. 2. Stimulant laxative. 3. Osmotic agent. 4. Pharmacological measures are not used in pediatric constipation.

3. Osmotic agent. A stool softener is the drug of choice because it will lead to easier evacuation.

Which is the best position for an 8-year-old who has just returned to the pediatric unit after an appendectomy for a ruptured appendix? 1. Semi-Fowler. 2. Prone. 3. Right side-lying. 4. Left side-lying.

3. The right side-lying position promotes comfort and allows the peritoneal cavity to drain.

Which is the best position for an 8-year-old who has just returned to the pediatric unit after an appendectomy for a ruptured appendix? 1. Semi-Fowler. 2. Prone. 3. Right side-lying. 4. Left side-lying.

3. The right side-lying position promotes comfort and allows the peritoneal cavity to drain.

The nurse is caring for a child with osteomyelitis who will be receiving high-dose intravenous antibiotic therapy for 3 to 4 weeks. What should the nurse plan to monitor? 1. Blood glucose level. 2. Thrombin times. 3. Urine glucose level. 4. Urine specific gravity.

4. Urine specific gravity.

21. A 10-year-old is being evaluated for possible appendicitis and complains of nausea and sharp abdominal pain in the right lower quadrant. An abdominal ultrasound is scheduled, and a blood count has been obtained. The child vomits, finds the pain relieved, and calls the nurse. Which should be the nurse's next action? 1. Cancel the ultrasound, and obtain an order for oral Zofran (ondansetron). 2. Cancel the ultrasound, and prepare to administer an intravenous bolus. 3. Prepare for the probable discharge of the patient. 4. Immediately notify the physician of the child's status.

4. Immediately notify the physician of the child's status. The physician should be notified immediately, as a sudden change or loss of pain often indicates a perforated appendix.

37. When teaching parents about osteosarcoma, the nurse knows instruction has been successful when a parent says that this type of cancer is common in which age group? 1. Infancy. 2. Toddlers. 3. School-aged children. 4. Adolescents.

4. Osteosarcoma is a common cancer of adolescents.

43. An infant with pyloric stenosis experiences excessive vomiting that can result in: a. hyperchloremia. b. hypernatremia. c. metabolic acidosis. d. metabolic alkalosis.

NS: D Infants with excessive vomiting are prone to metabolic alkalosis from the loss of hydrogen ions. Chloride ions and sodium are lost with vomiting. Metabolic alkalosis, not acidosis, is likely.

21. The nurse is caring for a 10-year-old in traction. While performing a skin assessment, the nurse notices that the skin over the calcaneus appears slightly red and irritated. Which action would the nurse take first? A. Reposition the child's foot on a pressure-reducing device. B. Apply lotion to his foot to maintain skin integrity. C. Make sure the skin is clean and dry. D. Gently massage his foot to promote circulation.

Rationale: The nurse's first action is to remove continuous pressure from this area. The other actions can help decrease the potential for skin breakdown, but the pressure must be relieved first.

26. The nurse is caring for a 3-year-old who had an appendectomy 2 days ago. The child has a fever of 101.8°F (38.8°C) and breath sounds are slightly diminished in the right lower lobe. Which action is most appropriate? 1. Teach the child how to use an incentive spirometer. 2. Encourage the child to blow bubbles. 3. Obtain an order for intravenous antibiotics. 4. Obtain an order for Tylenol (acetaminophen).

2. Encourage the child to blow bubbles. Blowing bubbles is a developmentally appropriate way to help the preschooler take deep breaths and cough.

A 9-year-old is given morphine for postoperative pain. As the nurse is assessing the client for pain 4 hours later, his mother leaves the room and the child begins to cry. The nurse's initial assessment of the child's pain is that he is: 1. Not in pain because the crying began after the mother leaves. 2. Less tolerant of pain because he is upset. 3. In pain because he is crying. 4. Not in pain because he was medicated 4 hours ago.

2. Less tolerant of pain because he is upset.

The nurse is caring for a child in Bryant's traction (see figure). The nurse should: 1. Adjust the weights on the legs until the but-tocks rest on the bed. 2. Provide frequent skin care. 3. Place a pillow under the buttocks. 4. Remove the elastic leg wraps every 8 hours for 10 minutes.

2. Provide frequent skin care.

The nurse knows that Nissen fundoplication involves which of the following? 1. The fundus of the stomach is wrapped around the inferior stomach, mimicking a lower esophageal sphincter. 2. The fundus of the stomach is wrapped around the inferior esophagus, mimicking a cardiac sphincter. 3. The fundus of the stomach is wrapped around the middle portion of the stomach, decreasing the capacity of the stomach. 4. The fundus of the stomach is dilated, decreasing the likelihood of reflux.

2. The fundus of the stomach is wrapped around the inferior esophagus, mimicking a cardiac sphincter. The Nissen fundoplication involves wrapping the fundus of the stomach around the inferior esophagus, creating a lower esophageal sphincter or cardiac sphincter.

30. A 9-year-old is in a spica cast and complains of pain 1 hour after receiving intravenous opioid analgesia. What should the nurse do first? 1. Give more pain medication. 2. Perform a neuromuscular assessment. 3. Call the surgeon for orders. 4. Tell the child to wait another hour for the medication to work.

2. The nurse looks for the source of the pain by performing a neuromuscular assessment.

8. Which classification of osteogenesis imperfecta (OI) is lethal in utero and in infancy? 1. Type I. 2. Type II. 3. Type III. 4. Type IV.

2. Type II is lethal in utero and in infancy because of multiple fractures and deformities and underdeveloped lungs.

The nurse is caring for a newborn who has just been diagnosed with tracheoesophageal fistula and is scheduled for surgery. Which should the nurse expect to do in the pre-operative period? 1. Keep the child in a monitored crib, obtain frequent vital signs, and allow the parents to visit but not hold their infant. 2. Administer intravenous fluids and antibiotics. 3. Place the infant on 100% oxygen via a non-rebreather mask. 4. Have the mother feed the infant slowly in a monitored area, stopping all feedings 4 to 6 hours before surgery.

2. Administer intravenous fluids and antibiotics. Intravenous fluids are administered to prevent dehydration because the infant is NPO. Intravenous antibiotics are administered to prevent pneumonia because aspiration of secretions is likely.

12. Which finding would lead the nurse to suspect that a child is experiencing moderate dehydration? A) Dusky extremities B) Tenting of skin C) Sunken fontanels D) Hypotension

Ans: C Feedback: A child with moderate dehydration would exhibit sunken fontanels. Severe dehydration would be characterized by dusky extremities, skin tenting, and hypotension.

When discussing with the child's mother the recommended diet for a child who is experiencing diarrhea, which teaching point made by the nurse is incorrect? 1) Avoid fruit juice and gelatin, because these are high in glucose. 2) We want your child to get back to his regular diet as quickly as possible. 3) We recommend the BRAT diet for children until they can resume their regular diet. 4) We really like to avoid the use of clear liquids in a child with diarrhea.

Answer: 3) The BRAT diet is not recommended anymore.

The nurse is conducting a staff in-service on casts. Which is an advantage to using a fiberglass cast instead of a plaster of Paris cast? a. Cheaper b. Dries rapidly c. Molds closely to body parts d. Smooth exterior

ANS: B A synthetic casting material dries in 5 to 30 minutes as compared with a plaster cast, which takes 10 to 72 hours to dry. Synthetic casts are more expensive and have a rough exterior, which may scratch surfaces. Plaster casts mold closer to body parts.

6. Parents call the clinic and report that their toddler has had acute diarrhea for 24 hours. The nurse should further ask the parents if the toddler has which associated factor that is causing the acute diarrhea? a. Celiac disease b. Antibiotic therapy c. Immunodeficiency d. Protein malnutrition

ANS: B Acute diarrhea is a sudden increase in frequency and change in consistency of stools and may be associated with antibiotic therapy. Celiac disease is a problem with gluten intolerance and may cause chronic diarrhea if not identified and managed appropriately. Immunodeficiency would occur with chronic diarrhea. Protein malnutrition or kwashiorkor causes chronic diarrhea from lowered resistance to infection.

Which is an appropriate nursing intervention when caring for a child in traction? a. Remove adhesive traction straps daily to prevent skin breakdown. b. Assess for tightness, weakness, or contractures in uninvolved joints and muscles. c. Provide active range-of-motion exercises to affected extremity three times a day. d. Keep the child in one position to maintain good alignment.

ANS: B Traction places stress on the affected bone, joint, and muscles. The nurse must assess for tightness, weakness, or contractures developing in the uninvolved joints and muscles. The adhesive straps should be released or replaced only when absolutely necessary. Active, passive, or active with resistance exercises should be carried out for the unaffected extremity only. Movement is expected with children. Each time the child moves, the nurse should check to ensure that proper alignment is maintained.

The parents of a 4-year-old ask the nurse how to manage their child's constipation. Select the nurse's best response. 1. "Add 2 ounces of apple or pear juice to the child's diet." 2. "Be sure your child eats a lot of fresh fruit such as apples and bananas." 3. "Encourage your child to drink more fluids." 4. "Decrease bulky foods such as whole-grain breads and rice."

3. "Encourage your child to drink more fluids." Increasing fluid consumption helps to decrease the hardness of the stool.

The nurse in the emergency department is caring for a 3-year-old child with a fractured humerus. The child is crying and screaming, "I hate you." Which of the following would be most appropriate? 1. Tell the parents they will need to wait out in the lobby. 2. Ask the charge nurse to assign this client to another nurse. 3. Reassure the parents that this a normal behavior under the circumstances. 4. Ask the parents to discipline the child so that the physician can treat her.

3. Reassure the parents that this a normal behavior under the circumstances.

The mother of a newborn asks the nurse why the infant has to nurse so frequently. Which is the best response? 1. Formula tends to be more calorically dense, and formula-fed babies require fewer feedings than breastfed babies. 2. The newborn's stomach capacity is small, and peristalsis is slow. 3. The newborn's stomach capacity is small, and peristalsis is more rapid than in older children. 4. Breastfed babies tend to take longer to complete a feeding than formula-fed babies.

3. The newborn's stomach capacity is small, and peristalsis is more rapid than in older children. The small-stomach capacity and rapid movement of fluid through the digestive system account for the need for small frequent feedings.

A 14-year-old has just had a plaster cast placed on his lower left leg. To provide safe cast care, the nurse should? 1. Petal the cast as soon as it is put on. 2. Keep the child in the same position for 24 hours until the cast is dry. 3. Use only the palms of the hand when handling the cast. 4. Notify the physician if the client complains of heat.

3. Use only the palms of the hand when handling the cast.

A 7-year-old child is seen in a clinic, and the health care provider documents a diagnosis of primary nocturnal enuresis. The nurse should provide which information to the parents? 1. Primary nocturnal enuresis does not respond to treatment. 2. Primary nocturnal enuresis is caused by a psychiatric problem. 3. Primary nocturnal enuresis requires surgical intervention to improve the problem. 4. Primary nocturnal enuresis is usually outgrown without therapeutic intervention.

4. Primary nocturnal enuresis occurs in a child who has never been dry at night for extended periods. The condition is common in children, and most children eventually outgrow bed-wetting without therapeutic intervention. The child is unable to sense a full bladder and does not awaken to void. The child may have delayed maturation of the central nervous system. The condition is not caused by a psychiatric problem.

The nurse is initiating nasogastric tube feedings in a child. What is the nurse's best action? 1. Microwave the formula. 2. Place the child in a prone position. 3. Encourage the child to point the head downward. 4. Position the child with the head slightly hyperflexed.

4. When initiating nasogastric tube feedings in a child, the child should be positioned so that the head is slightly hyperflexed or in a sniffing position with the nose pointed toward the ceiling. The formula should be warmed to room temperature, and a microwave should not be used.

The nurse is caring for a 4-year-old child immobilized by a fractured hip. Which complication should the nurse monitor related to the child's immobilization status? a. Metabolic rate increases b. Increased joint mobility leading to contractures c. Bone calcium increases, releasing excess calcium into the body (hypercalcemia) d. Venous stasis leading to thrombi or emboli formation

ANS: D The physiologic effects of immobilization, as a result of decreased muscle contraction, include venous stasis. This can lead to pulmonary emboli or thrombi. The metabolic rate decreases with immobilization. Loss of joint mobility leads to contractures. Bone demineralization with osteoporosis and hypercalcemia occur with immobilization.

The nurse uses the palms of the hands when handling a wet cast for which reason? a. To assess dryness of the cast b. To facilitate easy turning c. To keep the patient's limb balanced d. To avoid indenting the cast

ANS: D Wet casts should be handled by the palms of the hands, not the fingers, to avoid creating pressure points. Assessing dryness, facilitating easy turning, and keeping the patient's limb balanced are not reasons for using the palms of the hand rather than the fingers when handling a wet cast.

The nurse teaches the parents of an infant with developmental dysplasia of the hip how to handle their child in a Pavlik harness. Which of the following is most appropriate? 1.Fitting the diaper under the straps. 2.Leaving the harness off while the infant sleeps. 3.Checking for skin redness under straps every other day. 4.Putting powder on the skin under the straps every day.

1 The Pavlik harness is worn over a diaper. Knee socks are also worn to prevent the straps and foot and leg pieces from rubbing directly on the skin. For maximum results, the infant needs to wear the harness continuously. The skin should be inspected several times a day, not every other day, for signs of redness or irritation. Lotions and powders are to be avoided because they can cake and irritate the skin.

The nurse is interviewing the parents of a 6-year-old who has been experiencing constipation. Which could be a causative factor? Select all that apply. 1. Hypothyroidism. 2. Muscular dystrophy. 3. Myelomeningocele. 4. Drinks a lot of milk. 5. Active in sports.

1,2,3,4 1. Hypothyroidism can be a causative factor in constipation. 2. Weakened abdominal muscles can be seen in muscular dystrophy and can lead to constipation. 3. Myelomeningocele affects the innervation of the rectum and can lead to constipation. 4. Excessive milk consumption can lead to constipation.

The nurse is interviewing the parents of a 6-year-old who has been experiencing constipation. Which could be a causative factor? Select all that apply. 1. Hypothyroidism. 2. Muscular dystrophy. 3. Myelomeningocele. 4. Drinks a lot of milk. 5. Active in sports.

1,2,3,4 1. Hypothyroidism can be a causative factor in constipation. 2. Weakened abdominal muscles can be seen in muscular dystrophy and can lead to constipation. 3. Myelomeningocele affects the innervation of the rectum and can lead to constipation. 4. Excessive milk consumption can lead to constipation.

65. A nurse is caring for a 5-year-old who has a fracture of the tibia involving the growth plate. When providing information to the parents, the nurse should indicate that: 1. This is a serious injury that could cause long-term growth issues. 2. The fracture usually heals within 6 weeks without further complications. 3. The child will never be able to play contact sports. 4. Fractures involving the growth plate require pain medication.

1. Fractures of the growth plate are serious, as they can disrupt the growth process.

29. A 13-year-old just returned from surgery for scoliosis. Which nursing intervention(s) is/are appropriate in the first 24 hours? Select all that apply. 1. Assess for pain. 2. Logroll to change positions. 3. Get the teen to the bathroom 12 to 24 hours after surgery. 4. Check neurological status. 5. Monitor blood pressure.

1. General post-operative nursing interventions include assessing for pain. 2. Specific to scoliosis surgery, logrolling is the means of changing positions. 4. It is essential to check neurological status in a patient who just had scoliosis surgery. 5. General post-operative nursing interventions include assessing vital signs.

25. The nurse caring for a child with osteomyelitis assesses poor appetite. Which inter- vention(s) is/are most appropriate for this child? Select all that apply. 1. Offer high-calorie liquids. 2. Offer favorite foods. 3. Do not worry about intake, as appetite loss is expected. 4. Suggest intravenous removal to encourage oral intake. 5. Decrease pain medication that might cause nausea. 6. Offer frequent small meals.

1. High-calorie liquids are sometimes received better when the child has a poor appetite. 2. Offering favorite foods can sometimes tempt the child to eat, even with a poor appetite. 6. Small, frequent meals might increase daily caloric intake.

63. Nursing care of a child with a fractured extremity in whom there is suspected compartment syndrome includes which of the following? Select all that apply. 1. Assess pain. 2. Assess pulses. 3. Elevate extremity above the level of the heart. 4. Monitor capillary refill. 5. Provide pain medication as needed.

1. In a recent fracture, the nurse should assess pain and provide treatment. 2. Pain, pallor, and weak or absent pulses are all signs of compartment syndrome. 4. Weak or absent pulse is a sign of compartment syndrome, so monitoring capillary refill is important in assessment. 5. Pain, pallor, and weak or absent pulses are signs of compartment syndrome. Pain should be treated.

56. One nursing diagnosis for juvenile idiopathic arthritis (JIA) is impaired physical mobility. Select all that apply. 1. Give pain medication prior to ambulation. 2. Assist with range-of-motion activities. 3. Encourage the child to eat a high-fat diet. 4. Provide oxygen as necessary. 5. Use nonpharmacological methods, such as heat.

1. Providing pain medication prior to ambulation helps decrease pain during ambulation. 2. Children with JIA need to do range-of-motion exercises to prevent joint stiffness. 5. Using nonpharmacological methods such as heat helps with flexibility and pain.

When developing the teaching plan for parents using the Pavlik harness with their child, what should be the nurse's initial step? 1.Assessing the parents' current coping strategies. 2.Determining the parents' knowledge about the device. 3.Providing the parents with written instructions. 4.Giving the parents a list of community resources

2 Assessing the learner's knowledge level is the initial step in any teaching plan to promote the maximum amount of learning. This assessment also provides the nurse with a starting point for teaching. Assessing coping strategies can provide important information to the development of the teaching plan but is not the initial step. Giving parents written instructions or a list of community resources is appropriate once the parents' knowledge level has been determined and teaching has begun.

A child admitted to the hospital with a diagnosis of gastroenteritis and dehydration weighs 17 pounds 2 ounces (7.8 kg). The parents state that his preadmission weight was 18 pounds 4 ounces (8.3 kg). Based on weight alone, what type of dehydration does the nurse expect? 1. Mild dehydration 2. Moderate dehydration 3. Severe dehydration 4. Acute dehydration

2. Mild dehydration is a weight loss less than 5%; moderate dehydration is 5% to 10%; severe dehydration is greater than 10% weight loss. All types of dehydration are acute situations. The answer can be determined by calculating the percent of weight loss in dehydration. Because the math calculation determines more than a 5% weight loss but less than 10% weight loss, the correct answer is moderate dehydration. By calculating the percent of weight loss, the correct answer can be determined.

The nurse is caring for an infant who has been diagnosed with short bowel syndrome (SBS). The parent asks how the disease will affect the child. Select the nurse's best response. 1. "Because your child has a shorter intestine than most, your child will likely experience constipation and will need to be placed on a bowel regimen." 2. "Because your child has a shorter intestine than most, he will not be able to absorb all the nutrients and vitamins in food and will need to get nutrients in other ways." 3. "Unfortunately, most children with this diagnosis do not do very well." 4. "The prognosis and course of the disease have changed because hyperalimentation is available."

2. "Because your child has a shorter intestine than most, he will not be able to absorb all the nutrients and vitamins in food and will need to get nutrients in other ways."

The nurse is caring for a newborn with a cleft lip and palate. The mother states, "I will not be able to breastfeed my baby." Select the nurse's best response. 1. "It sounds like you are feeling discouraged. Would you like to talk about it?" 2. "Sometimes breastfeeding is still an option for babies with a cleft lip and palate. Would you like more information?" 3. "Although breastfeeding is not an option, you have the option of pumping your milk and then feeding it to your baby with a special nipple." 4. "We usually discourage breastfeeding babies with cleft lip and palate as it puts them at an increased risk for aspiration."

2. "Sometimes breastfeeding is still an option for babies with a cleft lip and palate. Would you like more information?" The breast can help fill in the cleft and help the infant create suction.

The nurse is caring for a newborn with a cleft lip and palate. The mother states, "I will not be able to breastfeed my baby." Select the nurse's best response. 1. "It sounds like you are feeling discouraged. Would you like to talk about it?" 2. "Sometimes breastfeeding is still an option for babies with a cleft lip and palate. Would you like more information?" 3. "Although breastfeeding is not an option, you have the option of pumping your milk and then feeding it to your baby with a special nipple." 4. "We usually discourage breastfeeding babies with cleft lip and palate as it puts them at an increased risk for aspiration."

2. "Sometimes breastfeeding is still an option for babies with a cleft lip and palate. Would you like more information?" The breast can help fill in the cleft and help the infant create suction.

Which child can be discharged without further evaluation? 1. A 2-year-old who has had 24 hours of watery diarrhea that has changed to bloody diarrhea in the past 12 hours. 2. A 2-year-old who had a relapse of one diarrhea episode after restarting a normal diet. 3. A 6-year-old who has been having vomiting and diarrhea for 2 days and has decreased urine output. 4. A 10-year-old who has just returned from a Scout camping trip and has had several episodes of diarrhea.

2. A 2-year-old who had a relapse of one diarrhea episode after restarting a normal diet. It is common for children to have a relapse of diarrhea after resuming a regular diet.

Which child can be discharged without further evaluation? 1. A 2-year-old who has had 24 hours of watery diarrhea that has changed to bloody diarrhea in the past 12 hours. 2. A 2-year-old who had a relapse of one diarrhea episode after restarting a normal diet. 3. A 6-year-old who has been having vomiting and diarrhea for 2 days and has decreased urine output. 4. A 10-year-old who has just returned from a Scout camping trip and has had several episodes of diarrhea.

2. A 2-year-old who had a relapse of one diarrhea episode after restarting a normal diet. It is common for children to have a relapse of diarrhea after resuming a regular diet.

24. Which is most important when teaching a parent about preventing osteomyelitis? 1. Parents can stop worrying about bone infection once their child reaches school age. 2. Parents need to clean open wounds thoroughly with soap and water. 3. Children will always get a fever if they have osteomyelitis. 4. Children should wear long pants when playing outside because their legs might get scratched.

2. Because bacteria from an open wound can lead to osteomyelitis, thorough cleaning with soap and water is the best prevention.

21. When a child is suspected of having osteomyelitis, the nurse can prepare the family to expect which of the following? Select all that apply. 1. Pain medication is contraindicated so that symptoms are not masked. 2. Blood cultures will be obtained. 3. Pus will be aspirated from the subperiosteum. 4. An intravenous line with antibiotics will be started. 5. Surgery will be necessary.

2. Blood cultures will be obtained. 3. Pus will be aspirated from the subperiosteum. 4. Antibiotics will be given via an intravenous line.

The nurse is caring for a 3-year-old who had an appendectomy 2 days ago. The child has a fever of 101.8°F (38.8°C) and breath sounds are slightly diminished in the right lower lobe. Which action is most appropriate? 1. Teach the child how to use an incentive spirometer. 2. Encourage the child to blow bubbles. 3. Obtain an order for intravenous antibiotics. 4. Obtain an order for Tylenol (acetaminophen).

2. Encourage the child to blow bubbles. Blowing bubbles is a developmentally appropriate way to help the preschooler take deep breaths and cough.

The nurse should teach the mother of a child who has a new cast for a fractured radius to do which of the following for the first few days at home? 1. Use a hair dryer to dry the cast more quickly. 2. Have the child refrain from strenuous activities. 3. Check movement and sensation of the child's fingers once a day. 4. Administer acetaminophen every 8 to 12 hours for discomfort.

2. Have the child refrain from strenuous activities.

The nurse is giving discharge instructions to the parent of a 1-month-old infant with tracheoesophageal fistula and a gastrostomy tube (GT). The nurse knows the mother understands the discharge teaching when she states: 1. "I will give my baby feedings through the GT but place liquid medications in the corner of the mouth to be absorbed." 2. "I will flush the GT with 2 ounces of water after each feeding to prevent the GT from clogging." 3. "I will clean the area around the GT with soap and water every day." 4. "I will place petroleum jelly around the GT if any redness develops."

3. "I will clean the area around the GT with soap and water every day." The area around the GT should be cleaned with soap and water to prevent an infection.

The nurse is caring for an infant with pyloric stenosis. The parent asks if any future children will likely have pyloric stenosis. Select the nurse's best response. 1. "You seem worried; would you like to discuss your concerns?" 2. "It is very rare for a family to have more than one child with pyloric stenosis." 3. "Pyloric stenosis can run in families. It is more common among males." 4. "Although there can be a genetic link, it is very unusual for girls to have pyloric stenosis."

3. "Pyloric stenosis can run in families. It is more common among males."

39. Which is most important to discuss with an adolescent who is going to have a leg amputation for osteosarcoma? 1. Pain. 2. Spirituality. 3. Body image. 4. Lack of coping.

3. Body image is a developmental issue for adolescents and influences their acceptance of themselves and by peers.

27. A 12-year-old diagnosed with scoliosis is to wear a brace for 23 hours a day. What is the most likely reason the child will not wear it for that long? 1. Pain from the brace. 2. Difficulty in putting the brace on. 3. Self-consciousness about appearance. 4. Not understanding what the brace is for.

3. Children this age are very conscious of their appearance and fitting in with their peers, so they might be very resistant to wearing a brace.

The nurse is caring for a 9-month-old with diarrhea secondary to rotavirus. The child has not vomited and is mildly dehydrated. Which is likely to be included in the discharge teaching? 1. Administer Imodium as needed. 2. Administer Kaopectate as needed. 3. Continue breastfeeding per routine. 4. The infant may return to day care 24 hours after antibiotics have been started.

3. Continue breastfeeding per routine. Breastfeeding is usually well tolerated and helps prevent death of intestinal villi and malabsorption.

Anticipating that a 3-year-old child in traction will have need for diversion, what should the nurse offer the child? Synthesize 1. A video game. 2. Blocks. 3. Hand puppets. 4. Marbles.

3. Hand puppets.

15. A 10-year-old with osteomyelitis has been on intravenous antibiotics for 48 hours. The child is allergic to amoxicillin. Vital signs are T 101.8°F (38.8°C), BP 100/60, P 96, R 24. Which is the primary reason for surgical treatment? 1. Young age. 2. Drug allergies. 3. Nonresponse to intravenous antibiotics. 4. Physician preference.

3. If a patient does not respond to an appropriate antibiotic withi 48 hours, surgery may be indicated.

The parents of a newborn diagnosed with a cleft lip and palate ask the nurse when their child's lip and palate will most likely be repaired. Select the nurse's best response. 1. "The palate and the lip are usually repaired in the first few weeks of life so that the baby can grow and gain weight." 2. "The palate and the lip are usually not repaired until the baby is approximately6 months old so that the mouth has had enough time to grow." 3. "The lip is repaired in the first few months of life, but the palate is not usually repaired until the child is 3 years old." 4. "The lip is repaired in the first few weeks of life, but the palate is not usually repaired until the child is 18 months old."

4. "The lip is repaired in the first few weeks of life, but the palate is not usually repaired until the child is 18 months old."

21. A 10-year-old is being evaluated for possible appendicitis and complains of nausea and sharp abdominal pain in the right lower quadrant. An abdominal ultrasound is scheduled, and a blood count has been obtained. The child vomits, finds the pain relieved, and calls the nurse. Which should be the nurse's next action? 1. Cancel the ultrasound, and obtain an order for oral Zofran (ondansetron). 2. Cancel the ultrasound, and prepare to administer an intravenous bolus. 3. Prepare for the probable discharge of the patient. 4. Immediately notify the physician of the child's status.

4. Immediately notify the physician of the child's status. The physician should be notified immediately, as a sudden change or loss of pain often indicates a perforated appendix.

The nurse is caring for a 3-month-old being evaluated for possible Hirschsprung disease. His parents call the nurse and show her his diaper containing a large amount of mucus and bloody diarrhea. The nurse notes that the infant is irritable and his abdomen appears very distended. Which should be the nurse's next action? 1. Reassure the parents that this is an expected finding and not uncommon. 2. Call a code for a potential cardiac arrest, and stay with the infant. 3. Immediately obtain all vital signs with a quick head-to-toe assessment. 4. Obtain a stool sample for occult blood.

3. Immediately obtain all vital signs with a quick head-to-toe assessment. All vital signs need to be evaluated because the child with enterocolitis can quickly progress to a state of shock. A quick head-to-toe assessment will allow the nurse to evaluate the child's circulatory system.

The parents of a child with a cleft palate are concerned and ask the nurse when the palate will be repaired. The nurse should plan to base the response on which information about cleft palate repair? 1. A cleft palate cannot be repaired in children. 2. Repair usually is performed by age 8 weeks. 3. Repair usually is performed by 2 months of age. 4. Repair usually is performed between 6 months and 2 years.

4. Cleft palate repair is individualized and is based on the degree of deformity and size of the child. Cleft palate repair usually is performed between 6 months and 2 years of age, depending on the preference of the health care provider. Early closure facilitates speech development. The remaining options are incorrect.

An infant is seen in the health care provider's office for complaints of frequent vomiting and spitting up after feedings. Findings indicate that the infant is not gaining weight, and gastroesophageal reflux is suspected. Which would the nurse anticipate being prescribed initially in the care of this child? 1. Administer omeprazole before feeding. 2. Place in prone position after each feeding. 3. Instruct parents to keep a log of feedings and any reflux present. 4. Administer predigested formula and feed small, frequent feedings.

4. For infants with frequent vomiting and spitting up, the diagnosis of gastroesophageal reflux should be considered. The initial action is to alter the formula to a predigested formula and feed small, frequent feedings.After the formula is changed, the family will be instructed to keep a log of feedings and any reflux with the new formula. Medication is not started until after the formula is changed. A prone position increases the risk of reflux and thus aspiration.

The nurse provides feeding instructions to a parent of an infant diagnosed with gastroesophageal reflux disease. Which instruction should the nurse give to the parent to assist in reducing the episodes of emesis? 1. Provide less frequent, larger feedings. 2. Burp the infant less frequently during feedings. 3. Thin the feedings by adding water to the formula. 4. Thicken the feedings by adding rice cereal to the formula.

4. Gastroesophageal reflux is backflow of gastric contents into the esophagus as a result of relaxation or incompetence of the lower esophageal or cardiac sphincter. Small, more frequent feedings with frequent burping often are prescribed in the treatment of gastroesophageal reflux. Feedings thickened with rice cereal may reduce episodes of emesis. If thickened formula is used, cross-cutting of the nipple may be required.

54. Which would the nurse teach a patient when NSAIDs are prescribed for treating juvenile idiopathic arthritis (JIA)? 1. Take with food. 2. Take on an empty stomach. 3. Blood levels are required for drug dosages. 4. Good oral hygiene is needed.

1. NSAIDs can cause gastric bleeding with long-term use; food helps to reduce the exposure of the drug on the stomach lining.

53. Which would the nurse teach an adolescent is a complication of corticosteroids used in the treatment of juvenile idiopathic arthritis (JIA)? 1. Fat loss. 2. Adrenal stimulation. 3. Immune suppression. 4. Hypoglycemia.

3. Steroids cause immune suppression, which is the reason behind its use in JIA; it reduces the body's attack on itself.

3. Which can occur in untreated developmental dysplasia of the hip (DDH)? Select all that apply. 1. Duck gait. 2. Pain. 3. Osteoarthritis in adulthood. 4. Osteoporosis in adulthood. 5. Increased flexibility of the hip joint in adulthood.

1, 2, 3. 1. Due to abnormal hip joint function, the patient's gait is stiff and waddling. 2. Due to abnormal femoral head placement, the patient may experience pain and decreased flexibility in adulthood. 3. Due to abnormal femoral head placement, the patient may experience osteoarthritis in the hip joint in adulthood.

7. When planning a rehabilitative approach for a child with osteogenesis imperfect (OI), the nurse should prevent which of the following? Select all that apply. 1. Positional contractures and deformities. 2. Bone infection. 3. Muscle weakness. 4. Osteoporosis. 5. Misalignment of lower extremity joints.

1, 3, 4, 5. 1. A long-term goal in caring for a child with OI is to prevent contractures and deformities. 3. A long-term goal in caring for a child with OI is to prevent muscle weakness. 4. A long-term goal in caring for a child with OI is to prevent osteoporosis. 5. A long-term goal in caring for a child with OI is to prevent misalignment of lower extremity joints.

Which child may need extra fluids to prevent dehydration? Select all that apply. 1. 7-day-old receiving phototherapy. 2. 6-month-old with newly diagnosed pyloric stenosis. 3. 2-year-old with pneumonia. 4. 2-year-old with full-thickness burns to the chest, back, and abdomen. 5. 13-year-old who has just started her menses.

1,2,3,4 1. The lights in phototherapy increase insensible fluid loss, requiring the nurse to monitor fluid status closely. 2. The infant with pyloric stenosis is likely to be dehydrated due to persistent vomiting. 3. A 2-year-old with pneumonia may have increased insensible fluid loss due to tachypnea associated with respiratory illness. The nurse needs to monitor fluid status cautiously because fluid overload can result in increased respiratory distress. 4. The child with a burn experiences extensive extracellular fluid loss and is at great risk for dehydration. The younger child is at greater risk due to greater proportionate body surface area.

One nursing diagnosis for juvenile idiopathic arthritis (JIA) is impaired physical mobility. Select all that apply. 1. Give pain medication prior to ambulation. 2. Assist with range-of-motion activities. 3. Encourage the child to eat a high-fat diet. 4. Provide oxygen as necessary. 5. Use nonpharmacological methods, such as heat.

1,2,5 1. Providing pain medication prior to ambulation helps decrease pain during ambulation. 2. Children with JIA need to do range of-motion exercises to prevent joint stiffness. 3. A high-fat diet is not helpful for mobility. 4. Oxygen is usually not necessary with the diagnosis of JIA. 5. Using nonpharmacological methods such as heat helps with flexibility and pain TEST-TAKING HINT: By understanding the disease process of JIA, the test taker will know what interventions are needed to help alleviate pain and disability.

The nurse is caring for an infant diagnosed with Hirschsprung disease. The mother states she is pregnant with a boy and wants to know if her new baby will likely have the disorder. Select the nurse's best response. 1. "Genetics play a small role in Hirschsprung disease, so there is a chance the baby will develop it as well." 2. "There is no evidence to support a genetic link, so it is very unlikely the baby will also have it." 3. "It is rarely seen in boys, so it is not likely your new baby will have Hirschsprung disease." 4. "Hirschsprung disease is seen only in girls, so your new baby will not be at risk."

1. "Genetics play a small role in Hirschsprung disease, so there is a chance the baby will develop it as well."

The parent of a child being evaluated for celiac disease asks the nurse why it is important to make dietary changes. Select the nurse's best response. 1. "The body's response to gluten causes damage to the mucosal cells in the intestine, leading to absorption problems." 2. "The body's response to consumption of anything containing gluten is to create special cells called villi, which leads to more diarrhea." 3. "The body's response to gluten causes the intestine to become more porous and hang on to more of the fat-soluble vitamins, leading to vitamin toxicity." 4. "The body's response to gluten causes damage to the mucosal cells, leading to malabsorption of water and hard, constipated stools."

1. "The body's response to gluten causes damage to the mucosal cells in the intestine, leading to absorption problems." The inability to digest protein leads to an accumulation of an amino acid that is toxic to the mucosal cells and villi, leading to absorption problems.

A child with newly diagnosed osteomyelitis has nausea and vomiting. The parent wishes to give the child ginger snaps to help control the nausea.The nurse should tell the parent: 1. "You can try them and see how he does." 2. "I will need to get an order." 3. "Your child needs medication for the vomiting." 4. "We discourage the use of home remedies in children."

1. "You can try them and see how he does."

4. A 4-month-old is brought to the emergency department with severe dehydration. The heart rate is 198, and her blood pressure is 68/38. The infant's anterior fontanel is sunken. The nurse notes that the infant does not cry when the intravenous line is inserted. The child's parents state that she has not "held anything down" in 18 hours. The nurse obtains a finger-stick blood sugar of 94. Which would the nurse expect to do immediately? 1. Administer a bolus of normal saline. 2. Administer a bolus of D10W. 3. Administer a bolus of normal saline with 5% dextrose added to the solution. 4. Offer the child an oral rehydrating solution such as Pedialyte.

1. Dehydration is corrected with the administration of an isotonic solution, such as normal saline or lactated Ringer solution. TEST-TAKING HINT: The test taker should immediately eliminate answers 2 and 3 as they both suggest administering glucose in bolus form, which is always contraindicated in pediatric patients. Answer 4 should be eliminated as the infant is severely dehydrated and not responding to painful stimulation, which is suggested by the lack of a cry on intravenous insertion.

44. A child is going to receive radiation for Ewing sarcoma. Which of the following is the best nursing intervention to prevent skin breakdown during therapy? 1. Advise the child to wear loose-fitting clothes to minimize irritation. 2. Advise the child to use emollients to prevent dry skin. 3. Apply cold packs nightly to reduce the warmth caused by the treatments. 4. Apply hydrocortisone to soothe itching from dry skin.

1. Loose clothing helps reduce irritation on the sensitive irradiated skin.

The nurse is caring for a newborn with esophageal atresia. When reviewing the mother's history, which would the nurse expect to find? 1. Maternal polyhydramnios. 2. Pregnancy lasting more than 38 weeks. 3. Poor nutrition during pregnancy. 4. Alcohol consumption during pregnancy.

1. Maternal polyhydramnios. Maternal polyhydramnios is present because the infant cannot swallow and absorb the amniotic fluid in utero.

A 10-year-old has 5 lb of Buck's extension traction on his left leg. The nurse should assess the child for which of the following? Select all that apply. 1. Dryness of the skin, by removing the foam wraps and boot. 2. Alignment of the shoulder, hips, and knees. 3. Frayed rope near pulleys. 4. Correct amount of traction weight on fracture. 5. Pressure on the coccyx.

2. Alignment of the shoulder, hips, and knees. 3. Frayed rope near pulleys. 4. Correct amount of traction weight on fracture. 5. Pressure on the coccyx.

55. Why are chemotherapeutic agents such as methotrexate and cyclophosphamide sometimes used to treat juvenile idiopathic arthritis (JIA)? 1. Effective against cancer-like JIA. 2. Affect the immune system. 3. Are similar to NSAIDs. 4. Are absorbed into the synovial fluid.

2. These drugs affect the immune system to reduce its ability to attack itself, as in the case of JIA.

A mother brings her 5-week-old infant to the health care clinic and tells the nurse that the child has been vomiting after meals. The mother reports that the vomiting is becoming more frequent and forceful. The nurse suspects pyloric stenosis and asks the mother which assessment question to elicit data specific to this condition? 1. "Are the stools ribbon-like, and is the infant eating poorly?" 2. "Does the infant suddenly become pale, begin to cry, and draw the legs up to the chest?" 3. "Does the vomit contain sour, undigested food without bile, and is the infant constipated?" 4. "Does the infant cry loudly and continuously during the evening hours but nurses or takes formula well?"

3. Vomiting undigested food that is not bile stained and constipation are classic symptoms of pyloric stenosis. Stools that are ribbon-like and a child who is eating poorly are signs of congenital megacolon (Hirschsprung's disease). An infant who suddenly becomes pale, cries out, and draws the legs up to the chest is demonstrating physical signs of intussusception. Crying during the evening hours, appearing to be in pain, eating well, and gaining weight are clinical manifestations of colic.

The nurse is caring for a 2-year-old child who was admitted to the pediatric unit for moderate dehydration due to vomiting and diarrhea. The child is restless, with periods of irritability. The child is afebrile with a heart rate of 148 and a blood pressure of 90/42. Baseline laboratory tests reveal the following: Na 152, Cl 119, and glucose 115. The parents state that the child has not urinated in 12 hours. After establishing a saline lock, the nurse reviews the physician's orders. Which order should the nurse question? 1. Administer a saline bolus of 10 mL/kg, which may be repeated if the child does not urinate. 2. Recheck serum electrolytes in 12 hours. 3. After the saline bolus, begin maintenance fluids of D5 1/4 NS with 10 mEq KCl/L. 4. Give clear liquid diet as tolerated.

3. After the saline bolus, begin maintenance fluids of D5 1/4 NS with 10 mEq KCl/L. Potassium is contraindicated because the child has not yet urinated. Potassium is not added to the maintenance fluid until kidney function has been verified.

36. When instructing a family about care of an orthosis, the nurse should emphasize which of the following? 1. Clean the brace with diluted bleach. 2. Dry the brace over a heater or in the sun. 3. Clean the brace weekly with mild soap and water. 4. Return the brace to the orthopedic surgeon for cleaning.

3. An orthosis should be cleaned weekly with mild soap and water.

3. A 4-month-old has had vomiting and diarrhea for 24 hours. The infant is fussy, and the anterior fontanel is sunken. The nurse notes the infant does not produce tears when crying. Which task will help confirm the diagnosis of dehydration? 1. Urinalysis obtained by bagged specimen. 2. Urinalysis obtained by sterile catheterization. 3. Analysis of serum electrolytes. 4. Analysis of cerebrospinal fluid.

3. Analysis of serum electrolytes. The analysis of serum electrolytes offers the most information and assists with the diagnosis of dehydration.

The parent of a 5-year-old states that the child has been having diarrhea for 24 hours, vomited twice 2 hours ago, and now claims to be thirsty. The parent asks what to offer the child because the child is refusing Pedialyte. Select the nurse's most appropriate response. 1. "You can offer clear diet soda such as Sprite and ginger ale." 2. "Pedialyte is really the best thing for your child, who, if thirsty enough, will eventually drink it." 3. "Pedialyte is really the best thing for your child. Allow your child some choice in the way to take it by offering small amounts in a spoon, medicine cup, or syringe." 4. "It really does not matter what your child drinks as long as it is kept down. Try offering small amounts of fluids in medicine cups."

3. Pedialyte is the first choice, as recom- mended by the American Academy of Pediatrics. Offering the child appropriate choices may allow the child to feel empowered and less likely to refuse the Pedialyte. Small, frequent amounts are usually better tolerated.

An emergency department nurse is performing an assessment on a child with a suspected diagnosis of intussusception. Which assessment question for the parents will elicit the most specific data related to this disorder? 1. "Does the child have any food allergies?" 2. "What do the bowel movements look like?" 3. "Has the child eaten any food in the last 24 hours?" 4. "Can you describe the type of pain that the child is experiencing?"

4. A report of severe colicky abdominal pain in a healthy, thriving child between 3 and 17 months of age is the classic presentation of intussusception. Typical behavior includes screaming and drawing the knees up to the chest. The remaining options are important aspects of a health history but are not specific to the diagnosis of intussusception.

A 16-month-old child is seen in the clinic for a checkup for the first time. The nurse notices that the toddler limps when walking. Which of the following would be appropriate to use when assessing this toddler for developmental dysplasia of the hip? 1.Ortolani's maneuver. 2.Barlow's maneuver. 3.Adam's position. 4.Trendelenburg's sign.

4. In a toddler, weight bearing causes the pelvis to tilt downward on the unaffected side instead of upward as it would normally. This is Trendelenburg's sign, and it indicates developmental dysplasia of the hip. Ortolani's maneuver is used during the neonatal period to assess developmental dysplasia of the hip in infants. With the infant quiet, relaxed, and lying on the back, the hips and knees are flexed at right angles. The knees are moved to abduction and pressure is exerted. If the femoral head moves forward, then it is dislocated. Barlow's maneuver is used to assess developmental dysplasia of the hip in infants. As the femur is moved into or out of the acetabulum, a "clunk" is heard, indicating dislocation. Adam's position is used to evaluate for structural scoliosis. The child bends forward with feet together and arms hanging freely or with palms together.

The nurse receives a call from the parent of a 10-month-old who has vomited three times in the past 8 hours. The parent describes the baby as playful and wanting to drink. The parent asks the nurse what to give the child. Select the nurse's best response. 1. "Replace the next feeding with regular water, and see if that is better tolerated." 2. "Do not allow your baby to eat any solids; give half the normal formula feeding, and see if that is better tolerated." 3. "Do not let your baby eat or drink anything for 24 hours to give the stomach a chance to rest." 4. "Give your child 1/2 ounce of Pedialyte every 10 minutes. If vomiting continues, wait an hour, and then repeat what you previously gave."

4. "Give your child 1/2 ounce of Pedialyte every 10 minutes. If vomiting continues, wait an hour, and then repeat what you previously gave." Offering small amounts of clear liquids is usually well tolerated. If the child vomits, make NPO to allow the stomach to rest and then restart fluids. The child in this scenario is described as playful and therefore does not appear to be at risk for dehydration.

The nurse is to receive a 4-year-old from the recovery room after an appendectomy. The parents have not seen the child since surgery and ask what to expect. Select the nurse's best response. 1. "Your child will be very sleepy, have an intravenous line in the hand, and have a nasal tube to help drain the stomach. If your child needs pain medication, it will be given intravenously." 2. "Your child will be very sleepy, have an intravenous line in the hand, and have white stockings to help prevent blood clots. If your child needs pain medication, we will give it intravenously or provide a liquid to swallow." 3. "Your child will be wide awake and will have an intravenous line in the hand. If your child needs pain medication, we will give it intravenously or provide a liquid to swallow." 4. "Your child will be very sleepy and have an intravenous line in the hand. If your child needs pain medication, we will give it int

4. "Your child will be very sleepy and have an intravenous line in the hand. If your child needs pain medication, we will give it intravenously." In the immediate post-operative period, the child is usually sleepy but can be roused. The child usually has an intravenous line for hydration and pain medication.

2. Which should the nurse stress to the parents of an infant in a Pavlik harness for treatment of developmental dysplasia of the hip (DDH)? 1. Put socks on over the foot pieces of the harness to help stabilize the harness. 2. Use lotions or powder on the skin to prevent rubbing of straps. 3. Remove harness during diaper changes for ease of cleaning diaper area. 4. Check under the straps at least two to three times daily for red areas.

4. Checking under straps frequently is suggested to prevent skin breakdown.

A child is diagnosed with juvenile idiopathic arthritis and is receiving several different medications listed in the medication administration record. Which agent would the nurse identify as being used to prevent disease progression? A)Aspirin B)Prednisone C)Ibuprofen D)Methotrexate

D. methotrexate Disease-modifying antirheumatic drugs (DMARDs), such as methotrexate, are necessary to prevent disease progression. Other agents, such as aspirin and ibuprofen, are helpful with pain relief. Prednisone helps for relief of inflammation.

Which would the nurse teach a patient when NSAIDs are prescribed for treating juvenile idiopathic arthritis (JIA)? 1. Take with food. 2. Take on an empty stomach. 3. Blood levels are required for drug dosages. 4. Good oral hygiene is needed.

1 1. NSAIDs can cause gastric bleeding with long-term use; food helps to reduce the exposure of the drug on the stomach lining. 2. NSAIDs can cause gastric bleeding with long-term use; food helps to reduce the exposure of the drug on the stomach lining. 3. NSAIDs do not require a blood level because they are available over the counter. 4. NSAIDs do not interfere with the oral cavity; however, other medications used for JIA cause oral ulcers. TEST-TAKING HINT: The test taker must know what NSAIDs are and that they are available over the counter

Which can occur in untreated developmental dysplasia of the hip (DDH)? Select all that apply. 1. Duck gait. 2. Pain. 3. Osteoarthritis in adulthood. 4. Osteoporosis in adulthood. 5. Increased flexibility of the hip joint in adulthood.

1, 2, 3. 1. Due to abnormal hip joint function, the client's gait is stiff and waddling. 2. Due to abnormal femoral head placement, the client may experience pain and decreased flexibility in adulthood. 3. Due to abnormal femoral head placement, the client may experience osteoarthritis in the hip joint in adulthood. 4. There is no increased risk for osteoporosis. 5. There is no increased flexibility of the hip joint in adulthood. TEST-TAKING HINT: If the test taker knows that untreated DDH leads to decreased smooth movement of hip joint, then answer 5 can be eliminated.

After a plaster cast has been applied to the arm of a child with a fractured right humerus, the nurse completes discharge teaching. The nurse should evaluate the teaching as successful when the mother agrees to seek medical advice if the child experiences which of the following? 1. Inability to extend the fingers on the right hand. 2. Vomiting after the cast is applied. 3. Coolness and dampness of the cast after 5hours. 4. Fussiness with complaints that the cast is heavy.

1. Inability to extend the fingers on the right hand.

13. Which would the nurse assess in a child diagnosed with osteomyelitis? Select all that apply. 1. Unwillingness to move affected extremity. 2. Severe pain. 3. Fever. 4. Previous closed fracture of an extremity. 5. Redness and swelling at the site.

1. Pain in an extremity leads to resistance to movement. 2. Pain is frequently severe in osteomyelitis. 3. Fever is present in the acute phase of the illness. 5. Redness and swelling occur because of the infection.

A child is admitted with a fracture of the femur and placed in skeletal traction. What should the nurse assess first? 1. The pull of traction on the pin. 2. The Ace bandage. 3. The pin sites for signs of infection. 4. The dressings for tightness.

1. The pull of traction on the pin.

The nurse is caring for an infant newly diagnosed with Hirschsprung disease. What does the nurse understand about this infant's condition? 1. There is a lack of peristalsis in the large intestine and an accumulation of bowel contents, leading to abdominal distention. 2. There is excessive peristalsis throughout the intestine, resulting in abdominal distention. 3. There is a small-bowel obstruction leading to ribbon-like stools. 4. There is inflammation throughout the large intestine, leading to accumulation of intestinal contents and abdominal distention.

1. There is a lack of peristalsis in the large intestine and an accumulation of bowel contents, leading to abdominal distention. In Hirschsprung disease, a portion of the large intestine has an area lacking in ganglion cells. This results in a lack of peristalsis as well as an accumulation of bowel contents and abdominal distention.

31. A 14-year-old with osteogenesis imperfecta (OI) is confined to a wheelchair. Which nursing interventions will promote normal development? Select all that apply. 1. Encourage participation in groups with teens who have disabilities or chronic illness. 2. Encourage decorating the wheelchair with stickers. 3. Encourage transfer of primary care to an adult provider at age 18 years. 4. Allow the teen to view the radiographs. 5. Help the teen set realistic goals for the future. 6. Discourage discussion of sexuality, as the child is not likely to date.

1. This patient is trying to become more independent and trying to fit in with the peer group. Encouraging socializing with peers who face similar challenges alleviates feelings of isolation. 2. Decorating the wheelchair encourages the patient to assume independence in self-care. 4. Allowing the patient to view radiographs encourages the patient to assume self-care. 5. Helping the patient set realistic goals for the future encourages independence.

46. A child with Ewing sarcoma is receiving chemotherapy and is experiencing severe nausea. The nurse has to administer Ativan at 0.04 mg/kg, and the child weighs 65 lb. What dose should the nurse administer?

1.2 mg. Change pounds to kilograms (2.2 lb = 1 kg: 65/2.2 = 29.5 kg). Then multiply kilograms by the dose of 0.04 mg/kg: 29.5 × 0.04 = 1.18 mg or round to 1.2 mg.

The mother of an 18-month-old child tells the clinic nurse that the child has been having some mild diarrhea and describes the child's stools as "mushy." The mother tells the nurse that the child is tolerating fluids and solid foods. The most appropriate suggestion regarding the child's diet would be to give the child which items? 1. Applesauce, bananas, wheat toast 2. Mashed potatoes with baked chicken 3. Gelatin, strained cabbage, and custard 4. Fluids only until the "mushy" stools stop

2. The continued feeding of a normal diet can prevent dehydration, reduce stool frequency and volume, and hasten recovery. Common foods that are especially well tolerated during diarrhea are bland but nutritional foods, including complex carbohydrates (rice, wheat, potatoes, cereals), yogurt containing live cultures, cooked vegetables, and lean meats. The foods in options 1 and 3 may worsen the diarrhea. Fluids only will affect nutritional status.

The nurse is caring for a 5-month-old infant with a diagnosis of intussusception. The infant has periods of irritability during which the knees are brought to chest and the infant cries, alternating with periods of lethargy. Vital signs are stable and within age-appropriate limits. The physician elects to give an enema. The parents ask the purpose of the enema. Select the nurse's most appropriate response. 1. "The enema will confirm the diagnosis. If the test result is positive, your child will need to have surgery to correct the intussusception." 2. "The enema will confirm the diagnosis. Although very unlikely, the enema may also help fix the intussusception so that your child will not immediately need surgery." 3. "The enema will help confirm the diagnosis and has a good chance of fixing the intussusception." 4. "The enema will help confirm the diagnosis and may temporarily fix the intussusception. If the bowel

3. "The enema will help confirm the diagnosis and has a good chance of fixing the intussusception." In most cases of intussusception in young children, an enema is successful in reducing the intussusception.

A preschooler with a fractured femur of the left leg in traction tells the nurse that his leg hurts. It is too early for pain medication. The nurse should: 1. Place a pillow under the child's buttocks to provide support. 2. Remove the weight from the left leg. 3. Assess the feet for signs of neurovascular impairment. 4. Reposition the pulleys so the traction is looser.

3. Assess the feet for signs of neurovascular impairment.

The nurse is caring for a 5-year-old who has just returned from having an appendectomy. Which is the optimal way to manage pain? 1. Intravenous morphine as needed. 2. Liquid Tylenol (acetaminophen) with codeine as needed. 3. Morphine administered through a PCA pump. 4. Intramuscular morphine as needed.

3. Morphine administered through a PCA pump. Morphine administered through a PCA pump offers the child control over managing pain. The PCA pump also has the benefit of offering a basal rate as well as an as-needed rate for optimal pain management.

Which would the nurse expect to be included in the diagnostic workup of a child with suspected celiac disease? 1. Obtain complete blood count and serum electrolytes. 2. Obtain complete blood count and stool sample; keep child NPO. 3. Obtain stool sample and prepare child for jejunal biopsy. 4. Obtain complete blood count and serum electrolytes; monitor child's response to gluten-containing diet.

3. Obtain stool sample and prepare child for jejunal biopsy. A stool sample for analysis of fat and a jejunal biopsy can confirm the diagnosis.

A child is diagnosed with chronic constipation that has been unresponsive to dietary and activity changes. Which pharmacological measure is most appropriate? 1. Natural supplements and herbs. 2. Stimulant laxative. 3. Osmotic agent. 4. Pharmacological measures are not used in pediatric constipation.

3. Osmotic agent. A stool softener is the drug of choice because it will lead to easier evacuation.

Which should the nurse stress to the parents of an infant in a Pavlik harness for treatment of developmental dysplasia of the hip (DDH)? 1. Put socks on over the foot pieces of the harness to help stabilize the harness. 2. Use lotions or powder on the skin to prevent rubbing of straps. 3. Remove harness during diaper changes for ease of cleaning diaper area. 4. Check under the straps at least two to three times daily for red areas.

4 1. Socks should be put on under the straps to prevent rubbing of the skin. The harness is stable if fitted correctly. 2. Lotions and powders tend to cake and irritate under the straps. Their use is not recommended. 3. The harness is not to be removed except in specific conditions and after instruction on removal and refitting. Diapering is easily done with the harness in place. 4. Checking under straps frequently is suggested to prevent skin breakdown. TEST-TAKING HINT: The test taker can eliminate answer 1 because the question is about skin redness and irritation, not harness fi t.

The nurse is teaching the parent of a child newly diagnosed with juvenile idiopathic arthritis (JIA). The nurse would evaluate the teaching as successful when the parent is able to say that the disorder is caused by the: 1. Breakdown of osteoclasts in the joint space causing bone loss. 2. Loss of cartilage in the joints. 3. Buildup of calcium crystals in joint spaces. 4. Immune-stimulated inflammatory response in the joint.

4 1. This is part of the normal breakdown and buildup of bone in the body. 2. This is part of the process for osteoarthritis. 3. This is the pathophysiology of calcium chondrosis. 4. JIA is caused by an immune response by the body on the joint spaces. TEST-TAKING HINT: The test taker must understand JIA is an immune-modulated disorder in which the body attacks itself, causing destruction of the joint spaces

The nurse is caring for a 14-year-old with celiac disease. The nurse knows that the patient understands the diet instructions by ordering which of the following meals? 1. Eggs, bacon, rye toast, and lactose-free milk. 2. Pancakes, orange juice, and sausage links. 3. Oat cereal, breakfast pastry, and nonfat skim milk. 4. Cheese, banana slices, rice cakes, and whole milk.

4. Cheese, banana slices, rice cakes, and whole milk.

The parent of a newborn asks, "Will my baby spit out the formula if it is too hot or too cold?" Select the nurse's best response. 1. "Babies have a tendency to reject hot fluids but not cold fluids, which could result in abdominal discomfort." 2. "Babies have a tendency to reject cold fluids but not hot fluids, which could result in esophageal burns." 3. "Your baby would most likely spit out formula that was too hot, but your baby could swallow some of it, which could result in a burn." 4. "Your baby is too young to be physically capable of spitting out fluids and will automatically swallow anything."

4. "Your baby is too young to be physically capable of spitting out fluids and will automatically swallow anything." Test taking hint: Swallowing is a reflex that is present until the age of 6 weeks. The test taker should eliminate answers 1, 2, and 3 as they suggest that the infant is capable of selectively rejecting fluids.

64. Which is the nurse's best explanation to the parent of a toddler who asks what a greenstick fracture is? 1. It is a fracture located in the growth plate of the bone. 2. Because children's bones are not fully developed, any fracture in a young child is called a greenstick fracture. 3. It is a fracture in which a complete break occurs in the bone, and small pieces of bone are broken off. 4. It is a fracture that does not go all the way through the bone.

4. It is a fracture that does not go all the way through the bone.

52. The nurse is teaching the parent of a child newly diagnosed with juvenile idiopathic arthritis (JIA). The nurse would evaluate the teaching as successful when the parent is able to say that the disorder is caused by the: 1. Breakdown of osteoclasts in the joint space causing bone loss. 2. Loss of cartilage in the joints. 3. Build-up of calcium crystals in joint spaces. 4. Immune-stimulated inflammatory response in the joint.

4. JIA is caused by an immune response by the body on the joint spaces.

4. The nurse is preparing to care for an infant returning from pyloromyotomy surgery. Which prescribed orders should the nurse anticipate implementing? (Select all that apply.) a. NPO for 24 hours b. Administration of analgesics for pain c. Ice bag to the incisional area d. IV fluids continued until tolerating PO e. Clear liquids as the first feeding

ANS: B, D, E Feedings are usually instituted soon after a pyloromyotomy surgery, beginning with clear liquids advancing to formula or breast milk as tolerated. IV fluids are administered until the infant is taking and retaining adequate amounts by mouth. Appropriate analgesics should be given around the clock because pain is continuous. Ice should not be applied to the incisional area as it vasoconstricts and would reduce circulation to the incisional area and impair healing.

5. An infant is brought to the emergency department with dehydration. Which physical assessment finding does the nurse expect? a. Weight gain b. Bradycardia c. Poor skin turgor d. Brisk capillary refill

ANS: C Clinical manifestations of dehydration include poor skin turgor, weight loss, lethargy, and tachycardia. The infant would have prolonged capillary refill, not brisk

A school nurse is conducting a staff in-service to other school nurses on idiopathic scoliosis. During which period of child development does idiopathic scoliosis become most noticeable? a. Newborn period b. When child starts to walk c. Preadolescent growth spurt d. Adolescence

ANS: C Idiopathic scoliosis is most noticeable during the preadolescent growth spurt. Idiopathic scoliosis is seldom apparent before age 10 years. Diagnosis usually occurs during the preadolescent growth spurt.

6. The nurse is caring for a 4-year-old boy who has undergone an appendectomy. The child is unwilling to use the incentive spirometer. Which approach would be most appropriate to elicit the child's cooperation? A) "Can you cough for me please?" B) "You must blow in this or you might get pneumonia." C) "If you don't try, I will have to get the healthcare provider." D) "Can you blow this cotton ball across the tray?"

Ans: D Feedback: Children are more likely to cooperate with interventions if play is involved. Encourage deep breathing by playing games. Asking the boy to cough is less likely to engage him. Telling the child he might get pneumonia is not age appropriate and is unhelpful. Threatening to call the healthcare provider is unhelpful and inappropriate. Remember, however, that the incentive spirometer works on the principle of the amount of air inhaled, not exhaled. Having the child take a deep breath prior to blowing the cotton ball is a beginning step.

27. After teaching the parents of a 6-year-old how to administer an enema, the nurse determines that the teaching was successful when they state that they will give how much solution to their child? A) 100 to 200 mL B) 200 to 300 mL C) 250 to 500 mL D) 500 to 1,000 mL

Ans: D Feedback: For a school-age child, typically 500 to 1,000 mL of enema solution is given. For an infant, 250 mL or less is used; for a toddler or preschooler, 250 to 500 mL is used.

42. A child is brought to the clinic after tripping over a rock. The child states "I twisted my ankle" and is given a diagnosis of a sprain. What intervention is most important for the nurse to include in the discharge instructions for this child? A. For the first 24 hours apply ice for 20 minutes and remove for 60 minutes B. Bedrest with leg elevated for 36 hours C. May take an NSAID for pain as prescribed D. Use compression dressing for 72 hours

Answer: A Rationale: A sprain results from twisting or a turning motion of the affected body part. Usually that is an ankle or a knee.The tendons and ligaments stretch excessively and may tear slightly. Edema, bruising and the inablility to bear weight are the most common symptoms. Interventions for care include RICE (rest, ice, compression, elevation), activity restrictions and/or splints or crutches. The most important intervention is the use of RICE. In this process the ice is applied for 20-30 minutes and then removed for 60 minutes. This can be done for up to 48 hours. This causes vasoconstriction to decrease the pain and swelling. Bedrest is not required, only limiting activities.Compression dressings, such as an elastic wrap are used, but there is no time limit as to how long they are needed. It depends upon the amount of swelling decreases. NSAIDs may be taken for pain if needed but the ice will produce a better pain relief.

The nurse is caring for the child with cleft lip and palate. Which of the following does the nurse understand as a complication of this disorder? Select all that apply: A) Heart malformation B) Otitis media C) Altered dentation D) Speech impediments E) Encopresis

Answer: B, C, and D. These are complications that can occur with cleft lip and palate. Others include feeding difficulties, aspiration, and hearing loss (related to ear infections).

The nurse is caring for a 4-month-old who has just had an isolated cleft lip repaired. Select the best position for the child in the immediate post-operative period. 1. Right side-lying. 2. Left side-lying. 3. Supine. 4. Prone.

3. Supine. The supine position is preferred because there is decreased risk of the infant rubbing the suture line.

The nurse is collecting data on an infant with a diagnosis of suspected Hirschsprung's disease. Which question to the mother will most specifically elicit information regarding this disorder? 1. "Does your infant have diarrhea?" 2. "Is your infant constantly vomiting?" 3. "Does your infant constantly spit up feedings?" 4. "Does your infant have foul-smelling, ribbon-like stools?"

4. Chronic constipation, beginning in the first month of life and resulting in pellet-like or ribbon stools that are foul-smelling, is a clinical manifestation of Hirschsprung's disease. Delayed passage or absence of meconium stool in the neonatal period is the primary sign. Bowel obstruction, especially in the neonatal period; abdominal pain and distension; and failure to thrive are also clinical manifestations. The remaining options are not specific clinical manifestations of this disorder.

The parents of a child just diagnosed with juvenile idiopathic arthritis (JIA) tell the nurse that the diagnosis frightens them because they know nothing about the prognosis. What should the nurse include when teaching the parents about the disease? 1.Half of affected children recover without joint deformity. 2.Many affected children go into long remissions but have severe deformities. 3.The disease usually progresses to crippling rheumatoid arthritis. 4.Most affected children recover completely within a few years.

1 In half of the children diagnosed with JIA, recovery occurs without joint deformity. Approximately one-third of the children will continue to have the disease into adulthood, and approximately one-sixth will experience severe, crippling deformities.

The nurse is caring for a 1-year-old child after cleft palate repair. On completion of feeding, the nurse should plan for which appropriate nursing action? 1. Rinsing the mouth with water 2. Cleaning the mouth with diluted hydrogen peroxide 3. Using a soft lemon and glycerin swab to clean the mouth 4. Using cotton swabs saturated with half-strength povidone-iodine to clean the mouth

1. After cleft palate repair, the mouth is rinsed with water after feedings to clean the palate repair site. Rinsing food and residual sugars from the suture line reduces the risk of infection. The remaining options are incorrect procedures. Hydrogen peroxide, lemon and glycerin, and povidone-iodine are not used because of their harmful effects on oral tissues and the suture site.

The nurse provides home care instructions to the parents of a child with celiac disease. The nurse should teach the parents to include which food item in the child's diet? 1. Rice 2. Oatmeal 3. Rye toast 4. Wheat bread

1. Celiac disease also is known as gluten enteropathy or celiac sprue and refers to intolerance to gluten, the protein component of wheat, barley, rye, and oats. The important factor to remember is that all wheat, rye, barley, and oats should be eliminated from the diet and replaced with corn, rice, or millet. Vitamin supplements—especially the fat-soluble vitamins, iron, and folic acid—may be needed to correct deficiencies. Dietary restrictions are likely to be lifelong.

The nurse has provided dietary instructions to the mother of a child with celiac disease. The nurse determines that the mother understands the instructions when the mother states to include which food in the child's diet? 1. Corn 2. Wheat cereal 3. Rye crackers 4. Oatmeal biscuits

1. Dietary management is the mainstay of treatment in celiac disease. All wheat, rye, barley, and oats should be eliminated from the diet and replaced with corn and rice. Vitamin supplements, especially fat-soluble vitamins and folate, may be needed in the early period of treatment to correct deficiencies. These are likely to be lifelong restrictions, although small amounts of grains may be tolerated after the ulcerations have healed.

The nurse is caring for an infant diagnosed with Hirschsprung disease. The mother states she is pregnant with a boy and wants to know if her new baby will likely have the disorder. Select the nurse's best response. 1. "Genetics play a small role in Hirschsprung disease, so there is a chance the baby will develop it as well." 2. "There is no evidence to support a genetic link, so it is very unlikely the baby will also have it." 3. "It is rarely seen in boys, so it is not likely your new baby will have Hirschsprung disease." 4. "Hirschsprung disease is seen only in girls, so your new baby will not be at risk."

1. "Genetics play a small role in Hirschsprung disease, so there is a chance the baby will develop it as well."

The parent of a child being evaluated for celiac disease asks the nurse why it is important to make dietary changes. Select the nurse's best response. 1. "The body's response to gluten causes damage to the mucosal cells in the intestine, leading to absorption problems." 2. "The body's response to consumption of anything containing gluten is to create special cells called villi, which leads to more diarrhea." 3. "The body's response to gluten causes the intestine to become more porous and hang on to more of the fat-soluble vitamins, leading to vitamin toxicity." 4. "The body's response to gluten causes damage to the mucosal cells, leading to malabsorption of water and hard, constipated stools."

1. "The body's response to gluten causes damage to the mucosal cells in the intestine, leading to absorption problems." The inability to digest protein leads to an accumulation of an amino acid that is toxic to the mucosal cells and villi, leading to absorption problems.

The nurse receives a call from the mother of a 6-month-old who describes her child as alternately sleepy and fussy. She states that her infant vomited once this morning and had two episodes of diarrhea. The last episode contained mucus and a small amount of blood. She asks the nurse what she should do. Select the nurse's best response. 1. "Your infant will need to have some tests in the emergency room to determine if anything serious is going on." 2. "Try feeding your infant in about 30 minutes; in the event of repeat vomiting, bring the infant to the emergency room for some tests and intravenous rehydration." 3. "Many infants display these symptoms when they develop an allergy to the formula they are receiving; try switching to a soy-based formula." 4. "Do not worry about the blood and mucus in the stool; it is not unusual for infants to have blood in their stools because their intestines are more sensitive."

1. "Your infant will need to have some tests in the emergency room to determine if anything serious is going on." The infant is displaying signs of intussusception. This is an emergency that needs to be evaluated to prevent ischemia and perforation.

62. A 6-year-old involved in a bicycle crash has a spleen injury and a right tibia/fibula fracture that has been casted. Which is/are an early sign(s) of compartment syndrome in this child? Select all that apply. 1. Edema. 2. Numbness. 3. Severe pain. 4. Weak pulse. 5. Anular rash.

1. Edema, numbness or tingling, and pain are early signs of compartment syndrome. 2. Edema, numbness or tingling, and pain are early signs of compartment syndrome. 4. A weak pulse is a late sign of compartment syndrome.

To meet the developmental needs of an 8-year-old child who is confined to home with osteomyelitis, what should the nurse include in the care plan? 1. Encouraging the child to communicate with schoolmates. 2. Encouraging the parents to stay with the child. 3. Allowing siblings to visit freely throughout the day. 4. Talking to the child about his interests twice daily.

1. Encouraging the child to communicate with schoolmates.

While assessing a 3-year-old child who has had an injury to the leg, has pain, and refuses to walk, the nurse notes that the child's left thigh is swollen. What should the nurse do next? 1. Assess the neurologic status of the toes. 2. Determine the circulatory status of the upper thigh. 3. Obtain the child's vital signs. 4. Notify the physician immediately.

1. Assess the neurologic status of the toes.

43. Which nursing diagnosis is most important for a child with Ewing sarcoma who will be undergoing chemotherapy? 1. Risk for fluid volume deficit. 2. Potential for chronic pain. 3. Risk for skin impairment. 4. Ineffective airway clearance.

1. Chemotherapy can cause nausea, vomiting, and possibly diarrhea, which contribute to fluid volume deficit.

42. The nurse is teaching an adolescent about Ewing sarcoma and indicates which as a common site? 1. Shaft. 2. Growth plate. 3. Ball of the femur. 4. Bone marrow.

1. Ewing sarcoma is a bone tumor that affects the shafts of long bones.

The nurse is caring for an infant newly diagnosed with Hirschsprung disease. What does the nurse understand about this infant's condition? 1. There is a lack of peristalsis in the large intestine and an accumulation of bowel contents, leading to abdominal distention. 2. There is excessive peristalsis throughout the intestine, resulting in abdominal distention. 3. There is a small-bowel obstruction leading to ribbon-like stools. 4. There is inflammation throughout the large intestine, leading to accumulation of intestinal contents and abdominal distention.

1. There is a lack of peristalsis in the large intestine and an accumulation of bowel contents, leading to abdominal distention. In Hirschsprung disease, a portion of the large intestine has an area lacking in ganglion cells. This results in a lack of peristalsis as well as an accumulation of bowel contents and abdominal distention.

Why are chemotherapeutic agents such as methotrexate (Trexall) and cyclophosphamide (Cytoxan) sometimes used to treat juvenile idiopathic arthritis (JIA)? 1. Are effective against cancer-like JIA. 2. Suppress the immune system. 3. Are similar to NSAIDs. 4. Are absorbed into the synovial fluid.

2 1. JIA is not a type of cancer. 2. These drugs affect the immune system to reduce its ability to attack itself, as in the case of JIA. 3. These medications are not the same as NSAIDs. 4. They are not absorbed into the synovial fluid to treat JIA; they suppress the immune system. TEST-TAKING HINT: There are some drugs that are used for other reasons outside of their usual use, such as chemotherapeutic agents. Many drugs have multiple uses.

The mother of a 4-year-old child with juvenile idiopathic arthritis (JIA) is worried that her child will have to stop attending preschool because of the illness. Which of the following responses by the nurse would be most appropriate? 1."It may be difficult for your child to attend school because of the side effects of the medications he will be prescribed." 2."Your child should be encouraged to attend school, but he'll need extra time to work out early morning stiffness." 3."You should keep your child at home from school whenever he experiences discomfort or pain in his joints." 4."Your child will probably need to wear splints and braces so that his joints will be supported properly."

2 Socialization is important for this preschool-age child, and activity is important to maintain function. Because children with JIA commonly experience most problems in the early morning after arising, they need more time to "warm up." Adverse effects may or may not occur. The child's normal routine needs to be maintained as much as possible. Although splints and braces may be needed, they are worn during periods of rest, not activity, to maintain function.

The mother asks the nurse about using a car seat for her toddler who is in a hip spica cast. The nurse should tell the mother: 1."You can use a seat belt because of the spica cast." 2."You will need a specially designed car seat for your toddler." 3."You can still use the car seat you already have." 4."You'll need to get a special release from the police so that a car seat won't be needed.

2 The toddler in a hip spica cast needs a specially designed car seat. The one that the mother already has will not be appropriate because of the need for the car seat to accommodate the cast and abductor bar

Nonsteroidal anti-inflammatory drugs are the first choice in treating a child with juvenile idiopathic arthritis. Which adverse effects should the nurse include in the teaching plan for the parents? Select all that apply. 1.Weight gain. 2.Abdominal pain. 3.Blood in the stool. 4.Folic acid deficiency. 5.Reduced blood clotting ability.

2,3,5 Adverse effects from nonsteroidal anti-inflammatory drugs include abdominal pain, blood in stool, and reduced clotting ability. Weight gain is common with corticosteroids. Folic acid deficiency is associated with methotrexate therapy

The nurse is caring for a child who was brought to the clinic complaining of severe abdominal pain and is suspected of having acute appendicitis. The child is lying on the examining table, with the knees pulled up toward the chest. What is the priority nursing action? 1. Collect urine sample for urinalysis 2. Perform a pain assessment using the FACES scale 3. Prepare the child for magnetic resonance imaging 4. Notify health care provider of white blood cell count above 10,000 mm3 (10 × 109/L)

2. A pain assessment is priority to assess for increased or reduced pain, which can indicate peritonitis. A reported sudden relief from pain may indicate perforation. If perforation occurs, increasing pain then ensues. Perforation can lead to peritonitis, an urgent condition to be treated immediately. Computed tomography has become the imaging technique of choice, although ultrasonography may also be helpful in diagnosing appendicitis. Urinalysis is analyzed to rule out a urinary tract infection but is not the priority of care. A white blood cell count above 10,000 mm3 (10 × 109/L) is an expected finding in appendicitis.

The nurse in the hospital is giving at-home feeding instructions to a family whose child is being discharged after being born with a cleft lip. Which statement by the mother would indicate that further teaching is indicated? 1. "I am so glad that I am able to breast-feed my baby." 2. "I must always feed my baby with a syringe and not use a nipple." 3. "I will feed my baby while sitting in a chair and holding her more upright." 4. "I will burp my baby very frequently so that she does not swallow a lot of air."

2. Infants with a cleft lip are fed using a special nipple. Therefore, although all the interventions relate to feeding, option 2 should be clarified with the family because if they fed the baby using a syringe, the child's oral needs for sucking will not be met. Breast-feeding is always an option and should be done unless the child is having difficulty. Most children with a small cleft lip can be breast-fed. Newborns should be burped frequently and fed in a somewhat upright position. These interventions are applicable to the child with a cleft lip as well.

45. A child with Ewing sarcoma is undergoing a limb salvage procedure. Which statement indicates the parents understand the procedure? 1. "Our child will have a bone graft to save the limb." 2. "Our child will need follow-up lengthening procedures." 3. "Our child will need shorter shirt sleeves." 4. "Our child will not need chemotherapy."

2. Limb salvage requires the lengthening procedures to encourage the bone to continue to grow so the child will not have a short limb.

A 12-year-old is having surgery to repair a fractured left femur. As a part of the preoperative safety procedures, the nurse should ask the client to: 1. Point to the area of the fracture. 2. Mark the location of the fracture with an "x"and sign his name. 3. Confirm with his parents that they have signed the operative permit. 4. State the surgery risks as understood from the surgeon.

2. Mark the location of the fracture with an "x"and sign his name.

38. A child with osteosarcoma is going to receive chemotherapy before surgery. Which statement by the parents indicates they understand the side effect of neutropenia? 1. "My child will be more at risk for diarrhea." 2. "My child will be more at risk for infection." 3. "My child's hair will fall out." 4. "My child will need to drink more."

2. Neutropenia makes a child more at risk for infection, because the immune system is compromised due to the chemotherapy.

The child in a new hip spica cast seems to be adjusting to the cast, except that after each meal the child tells the nurse that the cast is too tight. Which of the following should the nurse plan to do? 1. Administer a laxative prior to each meal. 2. Offer smaller, more frequent meals. 3. Give the child a mechanical soft diet. 4. Offer the child more fruits and grains.

2. Offer smaller, more frequent meals.

The parents of a 4-year-old ask the nurse how to manage their child's constipation. Select the nurse's best response. 1. "Add 2 ounces of apple or pear juice to the child's diet." 2. "Be sure your child eats a lot of fresh fruit such as apples and bananas." 3. "Encourage your child to drink more fluids." 4. "Decrease bulky foods such as whole-grain breads and rice."

3. "Encourage your child to drink more fluids." Increasing fluid consumption helps to decrease the hardness of the stool.

9. When counseling the parents of a child with osteogenesis imperfecta (OI), the nurse should include which of the following? Select all that apply. 1. Discourage future children because the condition is inherited. 2. Provide education about the child's physical limitations. 3. Give the parents a letter signed by the primary care provider explaining OI. 4. Provide information on contacting the Osteogenesis Imperfecta Foundation. 5. Encourage the parents to treat the child like their other children. 6. Encourage use of calcium to decrease risk of fractures.

2. The nurse should provide education about the child's physical limitations so that physical therapy and appropriate activity can be encouraged. 3. OI is frequently confused with child abuse. Carrying a letter stating that the child has OI and what that condition looks like can ease the stressors of an emergency department visit. 4. The Osteogenesis Imperfecta Foundation is an organization that can provide information and support for a family with a child with the condition.

28. A spinal curve of less than degrees that is nonprogressive does not require treatment for scoliosis.

20. A 20-degree spinal curve that is nonprogressive will not disfigure or interfere with normal functioning, so it is not treated with bracing or surgery.

Which would the nurse teach an adolescent is a complication of corticosteroids used in the treatment of juvenile idiopathic arthritis (JIA)? 1. Fat loss. 2. Adrenal stimulation. 3. Immune suppression. 4. Hypoglycemia.

3 1. Long-term corticosteroid use causes fat deposits, especially in the back, face, and trunk. 2. With the use of corticosteroids, there is adrenal suppression because the exogenous steroid causes the body to lower production of its own steroids. 3. Steroids cause immune suppression, which is the reason behind its use in JIA; it reduces the body's attack on itself. 4. Steroids cause hyperglycemia. TEST-TAKING HINT: The test taker must understand how immune system diseases work in order to know how treatments will be helpful. Consider how steroids work and their complications. Each answer listed, except the correct answer, is the opposite of the true side effects

When teaching the family of an older infant who has had a spica cast applied for developmental dysplasia of the hip, which information should the nurse include when describing the abduction stabilizer bar? 1.It can be adjusted to a position of comfort. 2.It is used to lift the child. 3.It adds strength to the cast. 4.It is necessary to turn the child.

3 The abduction bar is incorporated into the cast to increase the cast's strength and maintain the legs in alignment. The bar cannot be removed or adjusted, unless the cast is removed and a new cast is applied. The bar should never be used to lift or turn the client, because doing so may weaken the cast.

What should the nurse include when developing the teaching plan for the parents of a child with juvenile idiopathic arthritis who is being treated with naproxen (Naprosyn)? 1.Anti-inflammatory effect will occur in approximately 8 weeks. 2.Within 24 hours, the child will have anti-inflammatory relief. 3.The nurse should be called before giving the child any over-the-counter medications. 4.If a dose is forgotten or missed, that dose is not made up.

3 The first group of drugs typically prescribed is the nonsteroidal anti-inflammatory drugs, which include naproxen. Naproxen is included in only a few over-the-counter medications but aspirin is in several. The family should check with the nurse before giving any over-the-counter medications. Once therapy is started, it takes hours or days for relief from pain to occur. However, it takes 3 to 4 weeks for the anti-inflammatory effects to occur, including reduction in swelling and less pain with movement. The missed dose will need to be made up to maintain the serum level and to maintain therapeutic effectiveness of the drug

Which would the nurse expect to assess on a 3-week-old infant with developmental dysplasia of the hip (DDH)? Select all that apply. 1. Excessive hip abduction. 2. Femoral lengthening of an affected leg. 3. Asymmetry of gluteal and thigh folds. 4. Pain when lying prone. 5. Positive Ortolani test

3, 5. 1. In DDH, a newborn can have excessive hip adduction. 2. In DDH, an appearance of femoral shortening is frequently present on the affected side. 3. In DDH, asymmetrical thigh and gluteal folds are frequently present. 4. Infants do not experience pain from this condition. 5. The Ortolani maneuver moves a disclocated hip back into the socket with a distinct clunk. TEST-TAKING HINT: If the test taker knows that DDH decreases smooth movement of the femoral head, then answers 1 and 2 can be eliminated because they indicate increased movement of the femur.

An infant has just returned to the nursing unit after surgical repair of a cleft lip on the right side. The nurse should place the infant in which best position at this time? 1. Prone position 2. On the stomach 3. Left lateral position 4. Right lateral position

3. A cleft lip is a congenital anomaly that occurs as a result of failure of soft tissue or bony structure to fuse during embryonic development. After cleft lip repair, the nurse avoids positioning an infant on the side of the repair or in the prone position because these positions can cause rubbing of the surgical site on the mattress. The nurse positions the infant on the side lateral to the repair or on the back upright and positions the infant to prevent airway obstruction by secretions, blood, or the tongue. From the options provided, placing the infant on the left side immediately after surgery is best to prevent the risk of aspiration if the infant vomits.

A 12-year-old girl is admitted to the hospital with suspected appendicitis. What nursing interventions should be implemented preoperatively? 1. Applying a heating pad for 5-minute intervals as prescribed 2. Administering acetaminophen as needed for pain, as prescribed 3. Placing the adolescent in a fetal position, side-lying with legs drawn up to chest 4. Inserting a nasogastric tube and attaching it to low intermittent suction; measuring drainage as prescribed

3. A client with appendicitis is more comfortable when lying in what is traditionally known as the fetal position, with the legs drawn up toward the chest. This flexed positioning assists in decreasing the pain that comes with appendicitis by decreasing the pressure on the abdominal area. A heating pad is contraindicated because heat can lead to a ruptured appendix. Pain medications are not given to the client with acute appendicitis because they may mask the symptoms that accompany a ruptured appendix. A nasogastric tube may be necessary postoperatively for gastric decompression, or preoperatively if perforation occurs. There are no data in the question that support perforation.

The nurse is preparing a plan of care for an infant who will be returning from the recovery room following the surgical repair of a cleft lip located on the right side of the lip. On return from the recovery room, the nurse should plan to place the infant in which position? 1. Prone and flat 2. Supine and flat 3. On the left side 4. On the right side

3. Following cleft lip repair, the infant should be positioned supine or on the side lateral to the repair to prevent the suture line from contacting the bed linens. Immediately after surgery, it is best to place the infant on the left side rather than supine to prevent aspiration if the infant vomits.

The nurse is reviewing the health care provider's documentation in the record of a child admitted with a diagnosis of intussusception. The nurse expects to note that the health care provider has documented which manifestation? 1. Scleral jaundice 2. Projectile vomiting 3. Currant jelly stools 4. Pale-colored and hard stools

3. In the child with intussusception, bright red blood and mucus are passed through the rectum, resulting in what is commonly described as currant jelly stools. The child classically presents with severe abdominal pain that is crampy and intermittent, causing the child to draw the knees in to the chest. Vomiting may be present, but not projectile. Scleral jaundice and pale-colored, hard stools are not manifestations of this disorder.

A preschooler with a history of cleft palate repair comes to the clinic for a routine well-child checkup. To determine if this child is experiencing a long-term effect of cleft palate, which question should the nurse ask? 1. "Was the child recently treated for pneumonia?" 2. "Does the child play with an imaginary friend?" 3. "Is the child unresponsive when given directions?" 4. "Has the child had any difficulty swallowing food?"

3. Unresponsiveness may be an indication of hearing loss. A child who has a history of cleft palate should be routinely checked for hearing loss. Pneumonia and dysphagia are unrelated to cleft palate after repair. Having an imaginary friend is normal behavior for a preschool child. Many preschoolers with vivid imaginations have imaginary friends.

The nurse is caring for a 5-month-old infant with a diagnosis of intussusception. The infant has periods of irritability during which the knees are brought to chest and the infant cries, alternating with periods of lethargy. Vital signs are stable and within age-appropriate limits. The physician elects to give an enema. The parents ask the purpose of the enema. Select the nurse's most appropriate response. 1. "The enema will confirm the diagnosis. If the test result is positive, your child will need to have surgery to correct the intussusception." 2. "The enema will confirm the diagnosis. Although very unlikely, the enema may also help fix the intussusception so that your child will not immediately need surgery." 3. "The enema will help confirm the diagnosis and has a good chance of fixing the intussusception." 4. "The enema will help confirm the diagnosis and may temporarily fix the intussusception. If the bowel ret

3. "The enema will help confirm the diagnosis and has a good chance of fixing the intussusception." In most cases of intussusception in young children, an enema is successful in reducing the intussusception.

The nurse is caring for a 1-month-old term infant who experienced an anoxic episode at birth. The health-care team suspects that the infant is developing necrotizing enterocolitis (NEC). Which would the nurse expect to be included in the plan of care? 1. Immediately remove the feeding nasogastric tube (NGT) from the infant. 2. Obtain vital signs every 4 hours. 3. Prepare to administer antibiotics intravenously. 4. Change feedings to half-strength, administer slowly via a feeding pump.

3. Prepare to administer antibiotics intravenously. The feedings should immediately be stopped, but the NGT should be placed to allow decompression of the stomach. Vital signs should be obtained more frequently than every 4 hours because the infant is at high risk for peritonitis and sepsis. Intravenous antibiotics are administered to prevent or treat sepsis. Feedings are stopped immediately when a suspicion of NEC is present.

A preschool-age child with juvenile idiopathic arthritis (JIA) has become withdrawn, and the mother asks the nurse what she should do. Which of the following suggestions by the nurse would be most appropriate? 1.Introduce the child to other children her age who also have JIA. 2.Tell the mother to spend extra time with the child and less time with her other children. 3.Recommend that the mother send the child to see a counselor for therapy. 4.Encourage the mother to be supportive and understanding of the child.

4 Because the child is dealing with grief and loss associated with a chronic illness, parents need to be supportive and understanding. The child needs to feel valued and worthwhile. Introducing the child to others of the same age who also have JIA most probably would be ineffective because preschoolers are developmentally egocentric. Although the child needs to feel valued, the mother's spending more time with the child and less time with her other children is inappropriate because the child with JIA may experience secondary gain from the illness if the family interaction patterns are altered. Also, this action reinforces the child's withdrawal behavior. Psychological counseling is not needed at this time because the child's reaction is normal.

The father of a preschool-age child with a tentative diagnosis of juvenile idiopathic arthritis (JIA) asks about a test to definitively diagnose JIA. The nurse's response is based on knowledge of which of the following? 1.The latex fixation test is diagnostic. 2.An increased erythrocyte sedimentation rate is diagnostic. 3.A positive synovial fluid culture is diagnostic. 4.No specific laboratory test is diagnostic.

4 The nurse's response to the father is based on the knowledge that there is no definitive test for JIA. The latex fixation test, which is commonly used to diagnose arthritis in adults, is negative in 90% of children. The erythrocyte sedimentation rate may or may not be increased during active disease. This test identifies the presence of inflammation only. Synovial fluid cultures are done to rule out septic arthritis, not to diagnose JIA.

The nurse is assessing the infant shown in the figure. On observing the client from this angle, the nurse should document that this infant has which of the following? 1.Ortolani's "click." 2.Limited abduction. 3.Galeazzi's sign. 4.Asymmetric gluteal folds.

4 This infant with congenital hip dysplasia has asymmetric gluteal folds. The Ortolani "click" occurs when the nurse feels the femur sliding into the acetabulum with a "click." Limited abduction may be observed during an attempt to abduct the infant's thighs. Galeazzi's sign reveals femoral foreshortening and is observed by flexing the thighs.

The nurse is preparing an infant for surgery to treat Hirschsprung's disease. Which assessment finding is priority to identify and treat? 1. Vomiting and irritability 2. Malnourishment and lethargy 3. Abdominal distension and tenderness 4. Decreased blood pressure and tachycardia

4. Hirschsprung's disease is also known as congenital aganglionosis or megacolon. It is the result of an absence of ganglion cells in the rectum and, to varying degrees, upward in the colon. Nursing care management includes assessing for signs of enterocolitis, shock, fluid and electrolyte problems, and signs of bowel perforation. While all of the answer options are concerning, low blood pressure and tachycardia are signs of shock. Shock results in decreased perfusion and oxygenation to major organs and is the priority of care.

An infant is seen in the health care provider's office for complaints of projectile vomiting after feeding. Findings indicate that the child is fussy and is gaining weight but seems never to get enough to eat. Pyloric stenosis is suspected. Which prescription would the nurse anticipate having the highest priority in the care of this child? 1. Monitor intake and output. 2. Administer predigested formula. 3. Administer omeprazole before feeding. 4. Prepare the family for surgery for the child.

4. Infants with projectile vomiting after feeding that are fussy should be suspected of pyloric stenosis. The treatment for this diagnosis is surgery. The other options are treatment measures that may be prescribed for gastroesophageal reflux.

The nurse is caring for an 8-week-old male who has just been diagnosed with Hirschsprung disease. The parents ask what they should expect. Select the nurse's best response. 1. "It is really an easy disease to manage. Most children are placed on stool softeners to help with constipation until it resolves." 2. "A permanent stool diversion, called a colostomy, will be placed by the surgeon to bypass the narrowed area." 3. "Daily bowel irrigations will help your child maintain regular bowel habits." 4. "Although your child will require surgery, there are different ways to manage the disease, depending on how much bowel is involved."

4. "Although your child will require surgery, there are different ways to manage the disease, depending on how much bowel is involved." The aganglionic portion needs to be removed. Although most children have a temporary colostomy placed, many infants are able to bypass the colostomy and have the bowel immediately reattached.

More education about necrotizing enterocolitis (NEC) is needed in a nursing in-service when one of the participants states: 1. "Encouraging the mother to pump her milk for the feedings helps prevent NEC." 2. "Some sources state that the occurrence of NEC has increased because so many preterm infants are surviving." 3. "When signs of sepsis appear, the infant will likely deteriorate quickly." 4. "NEC occurs only in preemies and low-birth-weight infants."

4. "NEC occurs only in preemies and low-birth-weight infants." Although much more common in preterm and low-birth-weight infants, NEC is also seen in term infants as well.

The nurse is taking care of a 10-year-old child who has osteomyelitis. Which treatment plan is considered the primary method of treating osteomyelitis? a. Joint replacement b. Bracing and casting c. Intravenous antibiotic therapy d. Long-term corticosteroid therapy

ANS: C Osteomyelitis is an infection of the bone, most commonly caused by Staphylococcus aureus. The treatment of choice is antibiotics. Joint replacement, bracing and casting, and long-term corticosteroid therapy are not indicated for infectious processes.

The nurse is to receive a 4-year-old from the recovery room after an appendectomy. The parents have not seen the child since surgery and ask what to expect. Select the nurse's best response. 1. "Your child will be very sleepy, have an intravenous line in the hand, and have a nasal tube to help drain the stomach. If your child needs pain medication, it will be given intravenously." 2. "Your child will be very sleepy, have an intravenous line in the hand, and have white stockings to help prevent blood clots. If your child needs pain medication, we will give it intravenously or provide a liquid to swallow." 3. "Your child will be wide awake and will have an intravenous line in the hand. If your child needs pain medication, we will give it intravenously or provide a liquid to swallow." 4. "Your child will be very sleepy and have an intravenous line in the hand. If your child needs pain medication, we will give it intrave

4. "Your child will be very sleepy and have an intravenous line in the hand. If your child needs pain medication, we will give it intravenously." In the immediate post-operative period, the child is usually sleepy but can be roused. The child usually has an intravenous line for hydration and pain medication.

The nurse is caring for a 14-year-old with celiac disease. The nurse knows that the patient understands the diet instructions by ordering which of the following meals? 1. Eggs, bacon, rye toast, and lactose-free milk. 2. Pancakes, orange juice, and sausage links. 3. Oat cereal, breakfast pastry, and nonfat skim milk. 4. Cheese, banana slices, rice cakes, and whole milk.

4. Cheese, banana slices, rice cakes, and whole milk.

40. Which would be the best nursing intervention for a child with phantom pain after an amputation? 1. Tell the child that the pain does not exist. 2. Request a PCA pump from the physician for pain management. 3. Encourage the child to rub the stump. 4. Provide Elavil to help with pain.

4. Elavil is a medication for nerve pain that is helpful in relieving phantom pain.

Which can result from the bone demineralization associated with immobility? a. Osteoporosis b. Urinary retention c. Pooling of blood d. Susceptibility to infection

ANS: A Bone demineralization leads to a negative calcium balance, osteoporosis, pathologic fractures, extraosseous bone formation, and renal calculi. Urinary retention is secondary to the effect of immobilization on the urinary tract. Pooling of blood is a result of the cardiovascular effects of immobilization. Susceptibility to infection can result from the effects of immobilization on the respiratory and renal systems.

An adolescent with osteosarcoma is scheduled for a leg amputation in 2 days. The nurse's approach should include which action? a. Answering questions with straightforward honesty b. Avoiding discussing the seriousness of the condition c. Explaining that, although the amputation is difficult, it will cure the cancer d. Assisting the adolescent in accepting the amputation as better than a long course of chemotherapy

ANS: A Honesty is essential to gain the child's cooperation and trust. The diagnosis of cancer should not be disguised with falsehoods. The adolescent should be prepared for the surgery so he or she has time to reflect on the diagnosis and subsequent treatment. This allows questions to be answered. To accept the need for radical surgery, the child must be aware of the lack of alternatives for treatment. Amputation is necessary, but it will not guarantee a cure. Chemotherapy is an integral part of the therapy with surgery. The child should be informed of the need for chemotherapy and its side effects before surgery.

Which term is used to describe a type of fracture that does not produce a break in the skin? a. Simple b. Compound c. Complicated d. Comminuted

ANS: A If a fracture does not produce a break in the skin, it is called a simple, or closed, fracture. A compound, or open, fracture is one with an open wound through which the bone protrudes. A complicated fracture is one in which the bone fragments damage other organs or tissues. A comminuted fracture occurs when small fragments of bone are broken from the fractured shaft and lie in the surrounding tissue. These are rare in children.

12. A young child is brought to the emergency department with severe dehydration secondary to acute diarrhea and vomiting. Therapeutic management of this child should begin with: a. intravenous (IV) fluids. b. ORS. c. clear liquids, 1 to 2 ounces at a time. d. administration of antidiarrheal medication.

ANS: A In children with severe dehydration, IV fluids are initiated. ORS is acceptable therapy if the dehydration is not severe. Diarrhea is not managed by using clear liquids by mouth. These fluids have a high carbohydrate content, low electrolyte content, and high osmolality. Antidiarrheal medications are not recommended for the treatment of acute infectious diarrhea.

3. Which type of dehydration is defined as "dehydration that occurs in conditions in which electrolyte and water deficits are present in approximately balanced proportion"? a. Isotonic dehydration b. Hypotonic dehydration c. Hypertonic dehydration d. All types of dehydration in infants and small children

ANS: A Isotonic dehydration is the correct term for this definition and is the most frequent form of dehydration in children. Hypotonic dehydration occurs when the electrolyte deficit exceeds the water deficit, leaving the serum hypotonic. Hypertonic dehydration results from water loss in excess of electrolyte loss and is usually caused by a proportionately larger loss of water or a larger intake of electrolytes. This definition is specific to isotonic dehydration.

The nurse is preparing an adolescent with scoliosis for a Luque-rod segmental spinal instrumentation procedure. Which consideration should the nurse include? a. Nasogastric intubation and urinary catheter may be required. b. Ambulation will not be allowed for up to 3 months. c. Surgery eliminates the need for casting and bracing. d. Discomfort can be controlled with nonpharmacologic methods.

ANS: A Luque-rod segmental spinal instrumentation is a surgical procedure. Nasogastric intubation and urinary catheterization may be required. Ambulation is allowed as soon as possible. Depending on the instrumentation used, most patients walk by the second or third postoperative day. Casting and bracing are required postoperatively. The child usually has considerable pain for the first few days after surgery. Intravenous opioids should be administered on a regular basis.

A nurse is conducting a staff in-service on childhood cancers. Which is the primary site of osteosarcoma? a. Femur b. Humerus c. Pelvis d. Tibia

ANS: A Osteosarcoma is the most frequently encountered malignant bone cancer in children. The peak incidence is between ages 10 and 25 years. More than half occur in the femur. After the femur, most of the remaining sites are the humerus, tibia, pelvis, jaw, and phalanges.

45. The nurse is caring for a boy with probable intussusception. He had diarrhea before admission but, while waiting for administration of air pressure to reduce the intussusception, he passes a normal brown stool. Which nursing action is the most appropriate? a. Notify practitioner. b. Measure abdominal girth. c. Auscultate for bowel sounds. d. Take vital signs, including blood pressure.

ANS: A Passage of a normal brown stool indicates that the intussusception has reduced itself. This is immediately reported to the practitioner, who may choose to alter the diagnostic-therapeutic care plan. The first action would be to report the normal stool to the practitioner.

15. Which is a high-fiber food that the nurse should recommend for a child with chronic constipation? a. Popcorn b. Pancakes c. Muffins d. Ripe bananas

ANS: A Popcorn is a high-fiber food. Pancakes and muffins do not have significant fiber unless made with fruit or bran. Raw fruits, especially those with skins and seeds, other than ripe bananas, have high fiber.

A child is upset because, when the cast is removed from her leg, the skin surface is caked with desquamated skin and sebaceous secretions. Which should the nurse suggest to remove this material? a. Soak in a bathtub. b. Vigorously scrub leg. c. Apply powder to absorb material. d. Carefully pick material off leg.

ANS: A Simple soaking in the bathtub is usually sufficient for the removal of the desquamated skin and sebaceous secretions. It may take several days to eliminate the accumulation completely. The parents and child should be advised not to scrub the leg vigorously or forcibly remove this material because it may cause excoriation and bleeding. Oil or lotion, but not powder, may provide comfort for the child.

A young girl has just injured her ankle at school. In addition to calling the child's parents, the most appropriate, immediate action by the school nurse is to: a. apply ice. b. observe for edema and discoloration. c. encourage child to assume a position of comfort. d. obtain parental permission for administration of acetaminophen or aspirin.

ANS: A Soft-tissue injuries should be iced immediately. In addition to ice, the extremity should be rested, be elevated, and have compression applied. Observing for edema and discoloration, encouraging the child to assume a position of comfort, and obtaining parental permission for administration of acetaminophen or aspirin are not immediate priorities. The application of ice can reduce the severity of the injury.

49. Parents of a child undergoing an endoscopy to rule out peptic ulcer disease (PUD) from H. pylori ask the nurse, "If H. pylori is found will my child need another endoscopy to know that it is gone?" Which is the nurse's best response? a. "Yes, the only way to know the H. pylori has been eradicated is with another endoscopy." b. "We can collect a stool sample and confirm that the H. pylori has been eradicated." c. "A blood test can be done to determine that the H. pylori is no longer present." d. "Your child will always test positive for H. pylori because after treatment it goes into remission, but can't be completely eradicated."

ANS: B An upper endoscopy is the procedure initially performed to diagnose PUD. A biopsy can determine the presence of H. pylori. Polyclonal and monoclonal stool antigen tests are an accurate, noninvasive method to confirm H. pylori has been eradicated after treatment. A blood test can identify the presence of the antigen to this organism, but because H. pylori was already present it would not be as accurate as a stool sample to determine whether it has been eradicated. H. pylori can be treated and, once the treatment is complete, the stool sample can determine that it was eradicated.

Kristin, age 10 years, sustained a fracture in the epiphyseal plate of her right fibula when she fell off of a tree. When discussing this injury with her parents, the nurse should consider which statement? a. Healing is usually delayed in this type of fracture. b. Growth can be affected by this type of fracture. c. This is an unusual fracture site in young children. d. This type of fracture is inconsistent with a fall.

ANS: B Detection of epiphyseal injuries is sometimes difficult, but fractures involving the epiphysis or epiphyseal plate present special problems in determining whether bone growth will be affected. Healing of epiphyseal injuries is usually prompt. The epiphysis is the weakest point of the long bones. This is a frequent site of damage during trauma.

47. An infant with short bowel syndrome will be discharged home on total parenteral nutrition (TPN) and gastrostomy feedings. Nursing care should include: a. preparing family for impending death. b. teaching family signs of central venous catheter infection. c. teaching family how to calculate caloric needs. d. securing TPN and gastrostomy tubing under diaper to lessen risk of dislodgment.

ANS: B During TPN therapy, care must be taken to minimize the risk of complications related to the central venous access device, such as catheter infections, occlusions, or accidental removal. This is an important part of family teaching. The prognosis for patients with short bowel syndrome depends in part on the length of residual small intestine. It has improved with advances in TPN. Although parents need to be taught about nutritional needs, the caloric needs and prescribed TPN and rate are the responsibility of the health care team. The tubes should not be placed under the diaper due to risk of infection.

33. A newborn was admitted to the nursery with a complete bilateral cleft lip and palate. The physician explained the plan of therapy and its expected good results. However, the mother refuses to see or hold her baby. Initial therapeutic approach to the mother should be: a. restating what the physician has told her about plastic surgery. b. encouraging her to express her feelings. c. emphasizing the normalcy of her baby and the baby's need for mothering. d. recognizing that negative feelings toward the child continue throughout childhood.

ANS: B For parents, cleft lip and cleft palate deformities are particularly disturbing. The nurse must place emphasize not only the infant's physical needs but also the parents' emotional needs. The mother needs to be able to express her feelings before she can accept her child. Although the nurse will restate what the physician has told the mother about plastic surgery, it is not part of the initial therapeutic approach. As the mother expresses her feelings, the nurse's actions should convey to the parents that the infant is a precious human being. The nurse emphasizes the child's normalcy and helps the mother recognize the child's uniqueness. Maternal-infant attachment was not negatively affected at age 1 year.

40. Pyloric stenosis can best be described as: a. dilation of the pylorus. b. hypertrophy of the pyloric muscle. c. hypotonicity of the pyloric muscle. d. reduction of tone in the pyloric muscle.

ANS: B Hypertrophic pyloric stenosis occurs when the circumferential muscle of the pyloric sphincter becomes thickened, resulting in elongation and narrowing of the pyloric channel. Dilation of the pylorus, hypotonicity of the pyloric muscle, and reduction of tone in the pyloric muscle are not the definition of pyloric stenosis.

The nurse is caring for a preschool child immobilized by a spica cast. Which effect on metabolism should the nurse monitor on this child related to the immobilized status? a. Hypocalcemia b. Decreased metabolic rate c. Positive nitrogen balance d. Increased production of stress hormones

ANS: B Immobilization causes a decreased metabolic rate with slowing of all systems and a decreased food intake. Immobilization leads to hypercalcemia and causes a negative nitrogen balance secondary to muscle atrophy. A decreased production of stress hormones occurs with decreased physical and emotional coping capacity.

17. Enemas are ordered to empty the bowel preoperatively for a child with Hirschsprung disease. The enema solution should be: a. tap water. b. normal saline. c. oil retention. d. phosphate preparation.

ANS: B Isotonic solutions should be used in children. Saline is the solution of choice. Plain water is not used. This is a hypotonic solution and can cause rapid fluid shift, resulting in fluid overload. Oil-retention enemas will not achieve the "until clear" result. Phosphate enemas are not advised for children because of the harsh action of the ingredients. The osmotic effects of the phosphate enema can result in diarrhea, which can lead to metabolic acidosis.

35. A mother who intended to breastfeed has given birth to an infant with a cleft palate. Nursing interventions should include: a. giving medication to suppress lactation. b. encouraging and helping mother to breastfeed. c. teaching mother to feed breast milk by gavage. d. recommending use of a breast pump to maintain lactation until infant can suck.

ANS: B The mother who wishes to breastfeed may need encouragement and support because the defect does present some logistical issues. The nipple must be positioned and stabilized well back in the infant's oral cavity so that the tongue action facilitates milk expression. Because breastfeeding is an option, if the mother wishes to breastfeed, medications should not be given to suppress lactation. Because breastfeeding can usually be accomplished, gavage feedings are not indicated. The suction required to stimulate milk, absent initially, may be useful before nursing to stimulate the let-down reflex.

36. The nurse is caring for an infant whose cleft lip was repaired. Important aspects of this infant's postoperative care include: a. arm restraints, postural drainage, mouth irrigations. b. cleansing the suture line, supine and side-lying positions, arm restraints. c. mouth irrigations, prone position, cleansing suture line. d. supine and side-lying positions, postural drainage, arm restraints.

ANS: B The suture line should be cleansed gently after feeding. The child should be positioned on the back, on the side, or in an infant seat. Elbows are restrained to prevent the child from accessing the operative site. Postural drainage is not indicated. This would increase the pressure on the operative site when the child is placed in different positions. There is no reason to perform mouth irrigations, and the child should not be placed in the prone position where injury to the suture site can occur.

An adolescent with a fractured femur is in Russell's traction. Surgical intervention to correct the fracture is scheduled for the morning. Nursing actions should include which action? a. Maintaining continuous traction until 1 hour before the scheduled surgery b. Maintaining continuous traction and checking position of traction frequently c. Releasing traction every hour to perform skin care d. Releasing traction once every 8 hours to check circulation

ANS: B When the muscles are stretched, muscle spasm ceases and permits realignment of the bone ends. The continued maintenance of traction is important during this phase because releasing the traction allows the muscle's normal contracting ability to again cause malpositioning of the bone ends. Continuous traction must be maintained to keep the bone ends in satisfactory realignment. Releasing at any time, either 1 hour before surgery, once every hour for skin care, or once every 8 hours would not keep the fracture in satisfactory alignment.

The nurse is teaching a family how to care for their infant in a Pavlik harness to treat developmental dysplasia of the hip. Which should be included? a. Apply lotion or powder to minimize skin irritation. b. Remove harness several times a day to prevent contractures. c. Return to clinic every 1 to 2 weeks. d. Place diaper over harness, preferably using a superabsorbent disposable diaper that is relatively thin.

ANS: C Infants have a rapid growth pattern. The child needs to be assessed by the practitioner every 1 to 2 weeks for possible adjustments. Lotions and powders should not be used with the harness. The harness should not be removed, except as directed by the practitioner. A thin disposable diaper can be placed under the harness.

19. The nurse is explaining to a parent how to care for a school-age child with vomiting associated with a viral illness. Which action should the nurse include? a. Avoid carbohydrate-containing liquids. b. Give nothing by mouth for 24 hours. c. Brush teeth or rinse mouth after vomiting. d. Give plain water until vomiting ceases for at least 24 hours.

ANS: C It is important to emphasize the need for the child to brush the teeth or rinse the mouth after vomiting to dilute the hydrochloric acid that comes in contact with the teeth. Ad libitum administration of glucose-electrolyte solution to an alert child will help restore water and electrolytes satisfactorily. It is important to include carbohydrate to spare body protein and avoid ketosis.

2. A nurse is conducting an in-service on gastrointestinal disorders. The nurse includes that melena, the passage of black, tarry stools, suggests bleeding from which area? a. Perianal or rectal area b. Hemorrhoids or anal fissures c. Upper gastrointestinal (GI) tract d. Lower GI tract

ANS: C Melena is denatured blood from the upper GI tract or bleeding from the right colon. Blood from the perianal or rectal area, hemorrhoids, or lower GI tract would be bright red

10. Which therapeutic management should the nurse prepare to initiate first for a child with acute diarrhea and moderate dehydration? a. Clear liquids b. Adsorbents, such as kaolin and pectin c. Oral rehydration solution (ORS) d. Antidiarrheal medications such as paregoric

ANS: C ORS is the first treatment for acute diarrhea. Clear liquids are not recommended because they contain too much sugar, which may contribute to diarrhea. Adsorbents are not recommended. Antidiarrheals are not recommended because they do not get rid of pathogens

A nurse is conducting discharge teaching for parents of an infant with osteogenesis imperfecta (OI). Further teaching is indicated if the parents make which statement? a. "We will be very careful handling the baby." b. "We will lift the baby by the buttocks when diapering." c. "We're glad there is a cure for this disorder." d. "We will schedule follow-up appointments as instructed."

ANS: C The treatment for OI is primarily supportive. Although patients and families are optimistic about new research advances, there is no cure. The use of bisphosphonate therapy with IV pamidronate to promote increased bone density and prevent fractures has become standard therapy for many children with OI; however, long bones are weakened by prolonged treatment. Infants and children with this disorder require careful handling to prevent fractures. They must be supported when they are being turned, positioned, moved, and held. Even changing a diaper may cause a fracture in severely affected infants. These children should never be held by the ankles when being diapered but should be gently lifted by the buttocks or supported with pillows. Follow-up appointments for treatment with bisphosphonate can be expected.

42. The nurse is caring for an infant with suspected pyloric stenosis. Which clinical manifestation would indicate pyloric stenosis? a. Abdominal rigidity and pain on palpation b. Rounded abdomen and hypoactive bowel sounds c. Visible peristalsis and weight loss d. Distention of lower abdomen and constipation

ANS: C Visible gastric peristaltic waves that move from left to right across the epigastrium and weight loss are observed in pyloric stenosis. Abdominal rigidity and pain on palpation or a rounded abdomen and hypoactive bowel sounds are usually not present. The upper abdomen, not lower abdomen, is distended.

The nurse is caring for a preschool child with a cast applied recently for a fractured tibia. Which assessment findings indicate possible compartment syndrome? (Select all that apply.) a. Palpable distal pulse b. Capillary refill to extremity less than 3 seconds c. Severe pain not relieved by analgesics d. Tingling of extremity e. Inability to move extremity

ANS: C, D, E Indications of compartment syndrome are severe pain not relieved by analgesics, tingling of extremity, and inability to move extremity. A palpable distal pulse and capillary refill to the extremity less than 3 seconds are expected findings.

Which is a type of skin traction with legs in an extended position? a. Dunlop b. Bryant c. Russell d. Buck extension

ANS: D Buck extension traction is a type of skin traction with the legs in an extended position. It is used primarily for short-term immobilization, preoperatively with dislocated hips, for correcting contractures, or for bone deformities such as Legg-Calvé-Perthes disease. Dunlop traction is an upper-extremity traction used for fractures of the humerus. Bryant traction is skin traction with the legs flexed at a 90-degree angle at the hip. Russell traction uses skin traction on the lower leg and a padded sling under the knee. The combination of longitudinal and perpendicular traction allows realignment of the lower extremity and immobilizes the hips and knees in a flexed position.

11. A school-age child with diarrhea has been rehydrated. The nurse is discussing the child's diet with the family. Which statement by the parent would indicate a correct understanding of the teaching? a. "I will keep my child on a clear liquid diet for the next 24 hours." b. "I should encourage my child to drink carbonated drinks but avoid food for the next 24 hours." c. "I will offer my child bananas, rice, applesauce, and toast for the next 48 hours." d. "I should have my child eat a normal diet with easily digested foods for the next 48 hours."

ANS: D Easily digested foods such as cereals, cooked vegetables, and meats should be provided for the child. Early reintroduction of nutrients is desirable. Continued feeding or reintroduction of a regular diet has no adverse effects and actually lessens the severity and duration of the illness. Clear liquids and carbonated drinks have high carbohydrate content and few electrolytes. Caffeinated beverages should be avoided because caffeine is a mild diuretic. The BRAT diet has little nutritional value and is high in carbohydrates.

4. A nurse is admitting an infant with dehydration caused from water loss in excess of electrolyte loss. Which type of dehydration is this infant experiencing? a. Isotonic b. Isosmotic c. Hypotonic d. Hypertonic

ANS: D Hypertonic dehydration results from water loss in excess of electrolyte loss. This is the most dangerous type of dehydration. It is caused by feeding children fluids with high amounts of solute. Isotonic dehydration occurs in conditions in which electrolyte and water deficits are present in balanced proportion and is another term for isotonic dehydration. Hypotonic dehydration occurs when the electrolyte deficit exceeds the water deficit, leaving the serum hypotonic.

Which should cause a nurse to suspect that an infection has developed under a cast? a. Complaint of paresthesia b. Cold toes c. Increased respirations d. "Hot spots" felt on cast surface

ANS: D If hot spots are felt on the cast surface, they usually indicate infection beneath the area. This should be reported so that a window can be made in the cast to observe the site. The five Ps of ischemia from a vascular injury are pain, pallor, pulselessness, paresthesia, and paralysis. Paresthesia is an indication of vascular injury, not infection. Cold toes may be indicative of too tight a cast and need further evaluation. Increased respirations may be indicative of a respiratory tract infection or pulmonary emboli. This should be reported, and child should be evaluated.

44. Invagination of one segment of bowel within another is called: a. atresia. b. stenosis. c. herniation. d. intussusception

ANS: D Intussusception occurs when a proximal section of the bowel telescopes into a more distal segment, pulling the mesentery with it. The mesentery is compressed and angled, resulting in lymphatic and venous obstruction. Invagination of one segment of bowel within another is the definition of intussusception, not atresia, stenosis, or herniation.

16. Which therapeutic management treatment is implemented for children with Hirschsprung disease? a. Daily enemas b. Low-fiber diet c. Permanent colostomy d. Surgical removal of affected section of bowel

ANS: D Most children with Hirschsprung disease require surgical rather than medical management. Surgery is done to remove the aganglionic portion of the bowel, relieve obstruction, and restore normal bowel motility and function of the internal anal sphincter. Preoperative management may include enemas and low-fiber, high-calorie, high-protein diet, until the child is physically ready for surgery. The colostomy that is created in Hirschsprung disease is usually temporary.

30. The nurse is assessing a child with a possible fracture. What would the nurse identify as the most reliable indicator? A. Lack of spontaneous movement B. Point tenderness C. Bruising D. Inability to bear weight

Answer: B Rationale: Point tenderness is one of the most reliable indicators of a fracture in a child. Neglect of an extremity, inability to bear weight, bruising, erythema, and pain may be present, but these findings can also suggest other conditions.

18. An 8-year-old girl was diagnosed with a closed fracture of the radius at approximately 2 p.m. The fracture was reduced in the emergency department and her arm placed in a cast. At 11 p.m. her mother brings her back to the emergency department due to unrelenting pain that has not been relieved by the prescribed narcotics. Which action would be the priority? A. Notifying the doctor immediately B. Applying ice C. Elevating the arm D. Giving additional pain medication as ordered

Answer: A Rationale: The nurse should notify the doctor immediately because the girl's symptoms are the classic sign of compartment syndrome. Immediate treatment is required to prevent excessive swelling and to detect neurovascular compromise as quickly as possible. The ice should be removed and the arm brought below the level of the heart to facilitate whatever circulation is present. Giving additional pain medication will not help in this situation.

41. The school nurse has performed scoliosis screening. Based on this assessment, which children require the nurse to implement a referral to the healthcare provider? Select all that apply. A. The child with asymetric shoulder elevation B. The child with a limb length discrepancy C. The child with a lateral curve of the spine D. The child with a one-sided hump upon bending over E. The child who's sibling had scoliosis surgically corrected F. The child who has uneven balance

Answer: A, B, C, D Rationale: Scoliosis is defined by a lateral curve of the spine greater than 10 degrees.This curve causes displacement of the ribs. The nurse would first inspect the back in a standing position and note any asymetric shoulder elevation, the prominence of one scapula, an uneven curve at the waistline, or a rib hump on one side. While standing the nurse could also assess for leg length discrepancy and this could be measured. The nurse would then have the child bend over and observe for a pronounced hump on one side. The nurse should notify the parents and refer the child to the healthcare provider for evaluation if any of these symptoms are found. The sibling with a scoliosis repair would not be a concern unless it was known the family had a genetic diagnosis. Most scoliosis is idiopathic. Uneven balance is not a sign of scoliosis. The nurse would have to complete further assessments for this child.

The nurse understands that the young child is at a greater risk of developing fluid loss than an adult because of which of the following? Select all that apply: A) Greater body surface area B) Thinner skin C) Renal immaturity D) Higher likelihood of febrile illness E) Higher basic metabolic rate

Answer: A, C, D, and E.

You are taking care of an infant who has come back from having cleft lip and palate repair. The nurse would include all of the following in the plan of care except: A) Use of pacifier to prevent vigorous crying B) Holding, cuddling and rocking of infant C) Arm restraints or mummy restraint D) Placing infant in the supine position

Answer: A. It would be important to protect the palate operative site by avoiding putting items in the mouth that might disrupt the sutures such as suction catheters, spoons, straws, pacifiers, or plastic syringes. It would be important to keep the infant from rubbing the surgical sight. To prevent this the infant will be placed in the supine or side-lying position and arm restraints are often used. Holding, cuddling and rocking the infant can help soothe and comfort the infant after surgery.

25. The nurse is conducting a physical examination of a child with suspected developmental dysplasia of the hip. Which finding would help confirm this diagnosis? A. Abduction occurs to 75 degrees and adduction to within 30 degrees (with stable pelvis). B. A distinct "clunk" is heard with Barlow and Ortolani maneuvers. C. A high-pitched "click" is heard with hip flexion or extension. D. The thigh and gluteal folds are symmetric.

Answer: B Rationale: A distinct "clunk" while performing Barlow and Ortolani maneuvers is caused as the femoral head dislocates or reduces back in to the acetabulum. A higher-pitched "click" may occur with flexion or extension of the hip. This is a benign, adventitious sound that should not be confused with a true "clunk" when assessing for developmental dysplasia of the hip. Abduction to 75 degrees, adduction within 30 degrees, and symmetric thigh and gluteal folds are normal findings.

28. When teaching a group of parents about the skeletal development in children, what information is most helpful? A. The growth plate is made up of the epiphysis. B. A young child's bones commonly bend instead of break with an injury. C. The infant's skeleton has undergone complete ossification by birth. D. Children's bones have a thin periosteum and limited blood supply.

Answer: B Rationale: A young child's bones are more flexible and more porous with a lower mineral count than adults. Thus, bones will often bend rather than break when an injury occurs. The growth plate is composed of the epiphysis and physis. The infant's skeleton is not fully ossified at birth. Children's bones have a thick periosteum and an abundant blood supply.

34. A nurse is preparing a presentation for a parent group about musculoskeletal injuries. When describing a child's risk for this type of injury, the nurse integrates knowledge that bone growth occurs primarily in which area? A. Growth plate B. Epiphysis C. Physis D. Metaphysis

Answer: B Rationale: Growth of the bones occurs primarily in the epiphyseal region. This area is vulnerable and structurally weak. Traumatic force applied to the epiphysis during injury may result in fracture in that area of the bone. The growth plate refers to the combination of the epiphysis, the end of a long bone, and the physis, a cartilaginous area between the epiphysis and the metaphysis.

37. A nurse is providing instructions to the parents of a 3-month-old infant with developmental dysplasia of the hip who is being treated with a Pavlik harness. Which statement(s) by the parents demonstrates understanding of the instructions? Select all that apply. A. "We need to adjust the straps so that they are snug but not too tight." B. "We should change the diaper without taking our infant out of the harness." C. "We need to check the area behind our infant's knees for redness and irritation." D. "We need to send the harness to the dry cleaners to have it cleaned." E. "We need to call the health care provider if our infant is not able to actively kick the legs."

Answer: B, C, E Rationale: Instructions related to use of a Pavlik harness include changing the child's diaper while in the harness; checking the areas behind the knees and diaper area for redness, irritation, or breakdown; and calling the health care provider if the child is unable to actively kick the legs. The straps are not to be adjusted without checking with the health care provider first. The harness can be washed with mild detergent by hand and air dried. A hair dryer can be used to dry the harness but only if the air fluffing setting (no heat) is used.

31. An 8-year-old boy with a fractured forearm is to have a fiberglass cast applied. What information would the nurse include when teaching the child about the cast? A. The cast will take a day or two to dry completely. B. The edges will be covered with a soft material to prevent irritation. C. The child initially may experience a very warm feeling inside the cast. D. The child will need to keep his arm down at his side for 48 hours.

Answer: C Rationale: A fiberglass cast usually takes only a few minutes to dry and will cause a very warm feeling inside the cast. Therefore, the nurse needs to warn the child that this will occur. Fiberglass casts usually have a soft fabric edge so they usually do not cause skin rubbing at the edges and don't require petaling. The child should be instructed to elevate his arm above the level of the heart for the first 48 hours.

39. An 18-month-old was brought to the emergency department by her mother, who states, "I think she broke her arm." The child is sent for a radiograph to confirm the fracture. Additional assessment of the child leads the nurse to suspect possible child abuse. Which type of fracture would the radiograph most likely reveal? A. Plastic deformity B. Buckle fracture C. Spiral fracture D. Greenstick fracture

Answer: C Rationale: A spiral fracture is very rare in children. A spiral femoral or humeral fracture, particularly in a child younger than 2 years of age, should always be thoroughly investigated to rule out the possibility of child abuse. Plastic, buckle, and greenstick fractures are common in children and do not usually suggest child abuse.

23. The nurse is caring for a 14-year-old client in traction prior to surgery. The client has been in the hospital for 2 weeks and will require an additional 10 days in the hospital following surgery. The client states, "I feel isolated and I am refusing any more treatment." Which response by the nurse is most appropriate? A. "I know it is boring here, but the best place for you to remain immobile is the hospital." B. "I will see if you can have friends come spend a few nights with you." C. "Let's come up with things for you to do and see if your friends can come visit." D. "If you refuse further treatment, your condition will only get worse."

Answer: C Rationale: After 2 weeks in traction, an adolescent can become easily bored and isolated from usual peer interaction. The most helpful intervention would be to engage the help of the client to develop a list of books, games, movies, and other activities the client would enjoy. The nurse should also encourage visitation and phone calls from friends. Telling the client friends can come spend the night in the hospital is not most appropriate as minors are not typically encouraged to stay overnight. Telling the adolescent the condition will worsen if the client resists treatment is threatening and inappropriate.

27. The nurse is developing a teaching plan for a child who is to have his cast removed. What instruction would the nurse most likely include? A. Applying petroleum jelly to the dry skin B. Rubbing the skin vigorously to remove the dead skin C. Soaking the area in warm water every day D. Washing the skin with dilute peroxide and water

Answer: C Rationale: After a cast is removed, the child and family should be instructed to soak the area in warm water every day to help soften and remove the dry flaky skin. Moisturizing lotion, not petroleum jelly, should be applied to the skin. Vigorous rubbing would traumatize the skin and should be avoided. Warm soapy water, not dilute peroxide and water, should be used to wash the area.

19. The nurse is caring for an active 14-year-old boy who has recently been diagnosed with scoliosis. He is dismayed that a "jock" like himself could have this condition, and is afraid it will impact his spot on the water polo team. Which response by the nurse would best address the boy's concerns? A. "If you wear your brace properly, you may not need surgery." B. "The good news is that you have very minimal curvature of your spine." C. "Let's talk to another boy with scoliosis, who is winning trophies for his swim team." D. "Let's talk to the doctor about your treatment options."

Answer: C Rationale: Because this boy is concerned about limiting his participation in water polo and perceives scoliosis as a disease that does not affect "jocks," putting the child in contact with someone with the same problem would be helpful. Telling the adolescent about not needing surgery if he wears his brace or that his curvature is minimal may or may not be true in his case and thus would be false reassurance. Although these suggestions and also the suggestion about talking to the doctor about treatment options could be helpful by engaging his input in the treatment, these do not address his specific concerns about his body image.

A mother and 7 month old infant present to the pediatric clinic. The infant appears developmentally appropriate and healthy, but the mother tells you that she is exacerbated. She says yesterday her infant had been incessantly crying with vomiting and jelly-like stool. But now is fine. What is the nurse's first action? A) Determine prenatal status of the mother and child B) Prepare the child for immediate surgery C) Palpate the stomach for a mass D) Administer barium enema

Answer: C. The nurse would further assess the child. The nurse suspects this child to possibly have intussusception. A "sausage-like" mass in the upper mid-abdomen is a hallmark sign of intussusception. It may not be present at this time, but it would be important to assess for this finding. A barium enema is often used to treat this disorder. Surgery can also be used. The prenatal status of the mother/child would not be a priority assessment.

A mother and 7 month old infant present to the pediatric clinic. The infant appears developmentally appropriate and healthy, but the mother tells you that she is exacerbated. She says yesterday her infant had been incessantly crying with vomiting and jelly-like stool. But now, the infant appears fine. Which of the following GI disorders does the nurse suspect? A) Hypertrophic pyloric stenosis B) Celiac disease C) Intussusception D) Encopresis

Answer: C. Intussusception is when a proximal portion of the bowel "telescopes" into a more distal portion. This produces sudden onset, crampy abdominal pain accompanied by currant jelly stools, vomiting, crying/knee drawing up, and lethargy. This disorder is episodic and often the bowel will suddenly reduce down temporarily eliminating symptoms.

35. A group of nursing students are reviewing information about types of skin traction and skeletal traction. The students demonstrate understanding of this information when they identify which of these as a type of skeletal traction? A. Russell traction B. Bryant traction C. Buck traction D. Side arm 90-90 traction

Answer: D Rationale: Side arm 90-90 traction is a type of skeletal traction with force applied through a pin in the distal femur. Russell traction, Bryant traction, and Buck traction are types of skin traction.

When planning care for the infant diagnosed with cleft lip and palate, which action would the nurse take in relation to the priority nursing diagnosis for this child? A) Prevent the baby from vigorously crying B) Burp the baby well throughout feedings C) Temporarily refrain from having the baby breastfeed D) Encourage mother to use false palate covering when feeding baby

Answer: D. A false palate covering will help prevent the baby from aspirating while breastfeeding by providing a covering for the cleft palate. Adaptive nipples can also be used for this purpose. Burping the baby would be important to include in the plan of care, but would not be for the priority nursing diagnosis of risk for aspiration. It would not be necessary to have the baby refrain from breastfeeding. Preventing the baby from vigorously crying would be important postoperatively to prevent sutures from ripping.

The nurse is caring for a child who is having an anaphylactic reaction with bronchospasm. The nurse would expect to administer what medication for bronchospasm as ordered? A)Epinephrine B)Corticosteroid C)Albuterol D)Diphenhydramine

C. Albuterol The nurse would expect to administer bronchodilation inhalation treatment (albuterol) if bronchospasm is present. Epinephrine, diphenhydramine, and/or corticosteroids are administered to reverse the allergic process.

The nurse is assessing a child with pauciarticular-type juvenile idiopathic arthritis. What would the nurse expect to assess? A)Fever B)Rash C)Eye inflammation D)Splenomegaly

C. Eye inflammation With pauciarticular juvenile idiopathic arthritis, eye inflammation may be noted. Fever, rash, and enlarged spleen would be noted with systemic juvenile idiopathic arthritis.

Which exercise would the nurse suggest as most helpful to maintain mobility in a child with juvenile idiopathic arthritis? A)Jogging every other day B)Using a treadmill C)Swimming D)Playing basketball

C. Swimming Swimming is a particularly useful exercise to maintain joint mobility without placing pressure on the joints. Jogging, using a treadmill, and playing basketball would place pressure on the joints of the lower extremities.

A child is diagnosed with Hirschsprung's disease. The nurse is teaching the parents about the cause of the disease. Which statement, if made by the parent, supports that teaching was successful? 1. "Special cells are not present in the rectum, which caused the disease." 2. "The protein part of wheat, barley, rye, and oats is not being digested fully." 3. "The disease occurs from increased bowel motility that leads to spasm and pain." 4. "The disease occurs because of inability to tolerate sugar found in dairy products."

1. Hirschsprung's disease also is known as congenital aganglionosis or megacolon. It results from the absence of ganglion cells in the rectum and, to various degrees, up into the colon. Intolerance of wheat, barley, rye, and oats describes celiac disease. Intestinal spasm and pain describe irritable bowel syndrome. Irritability caused by dairy products describes lactose intolerance.

A child is suspected of suffering from intussusception. The nurse should be alert to which clinical manifestation of this condition? 1. Tender, distended abdomen 2. Presence of fecal incontinence 3. Incomplete development of the anus 4. Infrequent and difficult passage of dry stools

1. Intussusception is an invagination of a section of the intestine into the distal bowel. It is the most common cause of bowel obstruction in children aged 3 months to 6 years. A tender, distended abdomen is a clinical manifestation of intussusception. The presence of fecal incontinence describes encopresis. Encopresis generally affects preschool and school-aged children. Incomplete development of the anus describes imperforate anus, and this disorder is diagnosed in the neonatal period. The infrequent and difficult passage of dry stools describes constipation. Constipation can affect any child at any time, although the incidence peaks at age 2 to 3 years.

The nurse receives a call from the mother of a 6-month-old who describes her child as alternately sleepy and fussy. She states that her infant vomited once this morning and had two episodes of diarrhea. The last episode contained mucus and a small amount of blood. She asks the nurse what she should do. Select the nurse's best response. 1. "Your infant will need to have some tests in the emergency room to determine if anything serious is going on." 2. "Try feeding your infant in about 30 minutes; in the event of repeat vomiting, bring the infant to the emergency room for some tests and intravenous rehydration." 3. "Many infants display these symptoms when they develop an allergy to the formula they are receiving; try switching to a soy-based formula." 4. "Do not worry about the blood and mucus in the stool; it is not unusual for infants to have blood in their stools because their intestines are more sensitive."

1. "Your infant will need to have some tests in the emergency room to determine if anything serious is going on." The infant is displaying signs of intussusception. This is an emergency that needs to be evaluated to prevent ischemia and perforation.

Which is an accurate description of a Kasai procedure? 1. A palliative procedure in which the bile duct is attached to a loop of bowel to assist with bile drainage. 2. A curative procedure in which a connection is made between the bile duct and a loop of bowel to assist with bile drainage. 3. A curative procedure in which the bile duct is banded to prevent bile leakage. 4. A palliative procedure in which the bile duct is banded to prevent bile leakage.

1. A palliative procedure in which the bile duct is attached to a loop of bowel to assist with bile drainage.

Which should be included in the plan of care for a 14-month-old whose cleft palate was repaired 12 hours ago? Select all that apply. 1. Allow the infant to have familiar items of comfort such as a favorite stuffed animal and a "sippy" cup. 2. Once liquids have been tolerated, encourage a bland diet such as soup, Jell-O, and saltine crackers. 3. Administer pain medication on a regular schedule, as opposed to an as-needed schedule. 4. Use a Yankauer suction catheter on the infant's mouth to decrease the risk of aspiration of oral secretions. 5. When discharged, remove elbow restraints.

1. Allow the infant to have familiar items of comfort such as a favorite stuffed animal and a "sippy" cup. 3. Administer pain medication on a regular schedule, as opposed to an as-needed schedule. The child should not be allowed to use anything that creates suction in the mouth, such as pacifiers or straws. "Sippy" cups are acceptable. Pain medication should be administered regularly to avoid crying, which places stress on the suture line.

The nurse performing an admission assessment on a 2-year-old child who has been diagnosed with nephrotic syndrome notes that which most common characteristic is associated with this syndrome? 1. Hypertension 2. Generalized edema 3. Increased urinary output 4. Frank, bright red blood in the urine

2. Nephrotic syndrome is defined as massive proteinuria, hypoalbuminemia, hyperlipemia, and edema. Other manifestations include weight gain; periorbital and facial edema that is most prominent in the morning; leg, ankle, labial, or scrotal edema; decreased urine output and urine that is dark and frothy; abdominal swelling; and blood pressure that is normal or slightly decreased.

The nurse is assigned to care for a child who is scheduled for an appendectomy. Select the prescriptions that the nurse anticipates will be prescribed. Select all that apply. 1. Initiate an IV line. 2. Maintain an NPO status. 3. Administer a Fleet enema. 4. Administer intravenous antibiotics. 5. Administer preoperative medications. 6. Place a heating pad on the abdomen to decrease pain.

1.2.4.5 Appendicitis is an inflammation of the appendix. When the appendix becomes inflamed or infected, perforation may occur within a matter of hours, leading to peritonitis, sepsis, septic shock, and potential death. IV fluids would be started, and the child would be NPO while awaiting surgery. Usually antibiotics are administered because of the risk of perforation. Prescribed preoperative medications most likely would be administered on call to the operating room. In the preoperative period, enemas or laxatives should not be administered. Additionally, heat is not applied to the abdomen. Any of these interventions can cause rupture of the appendix and resultant peritonitis.

The nurse is providing discharge instructions to the mother of a child who had a cleft palate repair. Which statement should the nurse make to the mother? 1. "You should use a plastic spoon to feed the child." 2. "You need to use an orthodontic nipple on the child's bottle." 3. "You can allow the child to use a pacifier but only for 30 minutes at a time." 4. "You need to monitor the child's temperature for signs of infection using an oral thermometer."

2. An orthodontic nipple should be placed on the child's bottle, and the mother should be instructed to give the child baby food or baby food mixed with water. The mother should be instructed that straws, pacifiers, spoons, or fingers must be kept away from the child's mouth for 7 to 10 days after surgery. A pacifier should not be used for at least 2 weeks following the surgical repair. Additionally, the mother should be advised to avoid taking oral temperatures.

The nurse is caring for an infant with a diagnosis of bladder exstrophy. To protect the exposed bladder tissue, the nurse should plan which intervention? 1. Cover the bladder with petroleum jelly gauze. 2. Cover the bladder with a nonadhering plastic wrap. 3. Apply sterile distilled water dressings over the bladder mucosa. 4. Keep the bladder tissue dry by covering it with dry sterile gauze.

2. In bladder exstrophy, the bladder is exposed and external to the body. In this disorder, one must take care to protect the exposed bladder tissue from drying, while allowing the drainage of urine. This is accomplished best by covering the bladder with a nonadhering plastic wrap. The use of petroleum jelly gauze should be avoided because this type of dressing can dry out, adhere to the mucosa, and damage the delicate tissue when removed. Dry sterile dressings and dressings soaked in solutions (that can dry out) also damage the mucosa when removed.

The nurse admits a child to the hospital with a diagnosis of pyloric stenosis. On assessment, which data would the nurse expect to obtain when asking the parent about the child's symptoms? 1. Watery diarrhea 2. Projectile vomiting 3. Increased urine output 4. Vomiting large amounts of bile

2. In pyloric stenosis, hypertrophy of the circular muscles of the pylorus causes narrowing of the pyloric canal between the stomach and the duodenum. Clinical manifestations of pyloric stenosis include projectile vomiting, irritability, hunger and crying, constipation, and signs of dehydration, including a decrease in urine output.

The nurse is providing discharge instructions to the parents of a child who had an appendectomy for a ruptured appendix 5 days ago. The nurse knows that further education is required when the parent states: 1. "We will wait a few days before allowing our child to return to school." 2. "We will wait 2 weeks before allowing our child to return to sports." 3. "We will call the pediatrician's office if we notice any drainage around the wound." 4. "We will encourage our child to go for walks every day."

2. "We will wait 2 weeks before allowing our child to return to sports." The child should wait 6 weeks before returning to any strenuous activity.

Which manifestation suggests that an infant is developing necrotizing enterocolitis (NEC)? 1. Absorption of bolus orogastric feedings at a faster rate than previous feedings. 2. Bloody diarrhea. 3. Increased bowel sounds. 4. Appears hungry right before a scheduled feeding.

2. Bloody diarrhea. Bloody diarrhea can indicate that the infant has NEC.

The nurse is caring for a newborn with a suspected diagnosis of imperforate anus. The nurse monitors the infant, knowing that which is a clinical manifestation associated with this disorder? 1. Bile-stained fecal emesis 2. The passage of currant jelly-like stools 3. Failure to pass meconium stool in the first 24 hours after birth 4. Sausage-shaped mass palpated in the upper right abdominal quadrant

3. Imperforate anus is the incomplete development or absence of the anus in its normal position in the perineum. During the newborn assessment, this defect should be identified easily on sight. However, a rectal thermometer or tube may be necessary to determine patency if meconium is not passed in the first 24 hours after birth. Other assessment findings include absence or stenosis of the anal rectal canal, presence of an anal membrane, and an external fistula to the perineum. Options 1, 2, and 4 are findings noted in intussusception.

The nurse reviews the record of a newborn infant and notes that a diagnosis of esophageal atresia with tracheoesophageal fistula is suspected. The nurse expects to note which most likely sign of this condition documented in the record? 1. Incessant crying 2. Coughing at nighttime 3. Choking with feedings 4. Severe projectile vomiting

3. In esophageal atresia and tracheoesophageal fistula, the esophagus terminates before it reaches the stomach, ending in a blind pouch, and a fistula is present that forms an unnatural connection with the trachea. Any child who exhibits the "3 Cs"—coughing and choking with feedings and unexplained cyanosis—should be suspected to have tracheoesophageal fistula. Options 1, 2, and 4 are not specifically associated with tracheoesophageal fistula.

A child is hospitalized because of persistent vomiting. The nurse should monitor the child closely for which problem? 1. Diarrhea 2. Metabolic acidosis 3. Metabolic alkalosis 4. Hyperactive bowel sounds

3. Vomiting causes the loss of hydrochloric acid and subsequent metabolic alkalosis. Metabolic acidosis would occur in a child experiencing diarrhea because of the loss of bicarbonate. Diarrhea might or might not accompany vomiting. Hyperactive bowel sounds are not associated with vomiting.

The nurse is caring for a 3-month-old infant who has short bowel syndrome (SBS) and has been receiving total parenteral nutrition (TPN). The parents ask if their child will ever be able to eat. Select the nurse's best response. 1. "Children with SBS are never able to eat and must receive all of their nutrition in intravenous form." 2. "You will have to start feeding your child because children cannot be on TPN longer than 6 months." 3. "We will start feeding your child soon so that the bowel continues to receive stimulation." 4. "Your child will start receiving tube feedings soon but will never be able to eat by mouth."

3. "We will start feeding your child soon so that the bowel continues to receive stimulation." It is important to begin feedings as soon as the bowel is healed so that it receives stimulation and does not atrophy.

The nurse is caring for a 4-month-old who has just had an isolated cleft lip repaired. Select the best position for the child in the immediate post-operative period. 1. Right side-lying. 2. Left side-lying. 3. Supine. 4. Prone.

3. Supine. The supine position is preferred because there is decreased risk of the infant rubbing the suture line.

The nurse provided discharge instructions to the parents of a 2-year-old child who had an orchiopexy to correct cryptorchidism. Which statement by the parents indicates the need for further instruction? 1. "I'll check his temperature." 2. "I'll give him medication so he'll be comfortable." 3. "I'll check his voiding to be sure there's no problem." 4. "I'll let him decide when to return to his play activities."

4. Cryptorchidism is a condition in which 1 or both testes fail to descend through the inguinal canal into the scrotal sac. Surgical correction may be necessary. All vigorous activities should be restricted for 2 weeks after surgery to promote healing and prevent injury. This prevents dislodging of the suture, which is internal. Normally, 2-year-olds want to be active; allowing the child to decide when to return to his play activities may prevent healing and cause injury. The parents should be taught to monitor the temperature, provide analgesics as needed, and monitor the urine output.

The clinic nurse reviews the record of an infant and notes that the health care provider has documented a diagnosis of suspected Hirschsprung's disease. The nurse reviews the assessment findings documented in the record, knowing that which sign most likely led the mother to seek health care for the infant? 1. Diarrhea 2. Projectile vomiting 3. Regurgitation of feedings 4. Foul-smelling ribbon-like stools

4. Hirschsprung's disease is a congenital anomaly also known as congenital aganglionosis or aganglionic megacolon. It occurs as the result of an absence of ganglion cells in the rectum and other areas of the affected intestine. Chronic constipation beginning in the first month of life and resulting in pellet-like or ribbon-like stools that are foul-smelling is a clinical manifestation of this disorder. Delayed passage or absence of meconium stool in the neonatal period is also a sign. Bowel obstruction, especially in the neonatal period; abdominal pain and distention; and failure to thrive are also clinical manifestations. Options 1, 2, and 3 are not associated specifically with this disorder.

The nurse is reviewing a treatment plan with the parents of a newborn with hypospadias. Which statement by the parents indicates their understanding of the plan? 1. "Caution should be used when straddling the infant on a hip." 2. "Vital signs should be taken daily to check for bladder infection." 3. "Catheterization will be necessary when the infant does not void." 4. "Circumcision has been delayed to save tissue for surgical repair."

4. Hypospadias is a congenital defect involving abnormal placement of the urethral orifice of the penis. In hypospadias, the urethral orifice is located below the glans penis along the ventral surface. The infant should not be circumcised because the dorsal foreskin tissue will be used for surgical repair of the hypospadias. Options 1, 2, and 3 are unrelated to this disorder.

The nurse is preparing to care for a child with a diagnosis of intussusception. The nurse reviews the child's record and expects to note which sign of this disorder documented? 1. Watery diarrhea 2. Ribbon-like stools 3. Profuse projectile vomiting 4. Bright red blood and mucus in the stools

4. Intussusception is a telescoping of 1 portion of the bowel into another. The condition results in an obstruction to the passage of intestinal contents. A child with intussusception typically has severe abdominal pain that is crampy and intermittent, causing the child to draw in the knees to the chest. Vomiting may be present, but is not projectile. Bright red blood and mucus are passed through the rectum and commonly are described as currant jelly-like stools. Watery diarrhea and ribbon-like stools are not manifestations of this disorder.

The nurse is caring for an 8-week-old male who has just been diagnosed with Hirschsprung disease. The parents ask what they should expect. Select the nurse's best response. 1. "It is really an easy disease to manage. Most children are placed on stool softeners to help with constipation until it resolves." 2. "A permanent stool diversion, called a colostomy, will be placed by the surgeon to bypass the narrowed area." 3. "Daily bowel irrigations will help your child maintain regular bowel habits." 4. "Although your child will require surgery, there are different ways to manage the disease, depending on how much bowel is involved."

4. "Although your child will require surgery, there are different ways to manage the disease, depending on how much bowel is involved." The aganglionic portion needs to be removed. Although most children have a temporary colostomy placed, many infants are able to bypass the colostomy and have the bowel immediately reattached.

The nurse receives a call from the parent of a 10-month-old who has vomited three times in the past 8 hours. The parent describes the baby as playful and wanting to drink. The parent asks the nurse what to give the child. Select the nurse's best response. 1. "Replace the next feeding with regular water, and see if that is better tolerated." 2. "Do not allow your baby to eat any solids; give half the normal formula feeding, and see if that is better tolerated." 3. "Do not let your baby eat or drink anything for 24 hours to give the stomach a chance to rest." 4. "Give your child 1/2 ounce of Pedialyte every 10 minutes. If vomiting continues, wait an hour, and then repeat what you previously gave."

4. "Give your child 1/2 ounce of Pedialyte every 10 minutes. If vomiting continues, wait an hour, and then repeat what you previously gave." Offering small amounts of clear liquids is usually well tolerated. If the child vomits, make NPO to allow the stomach to rest and then restart fluids. The child in this scenario is described as playful and therefore does not appear to be at risk for dehydration.

The parents of a newborn diagnosed with a cleft lip and palate ask the nurse when their child's lip and palate will most likely be repaired. Select the nurse's best response. 1. "The palate and the lip are usually repaired in the first few weeks of life so that the baby can grow and gain weight." 2. "The palate and the lip are usually not repaired until the baby is approximately 6 months old so that the mouth has had enough time to grow." 3. "The lip is repaired in the first few months of life, but the palate is not usually repaired until the child is 3 years old." 4. "The lip is repaired in the first few weeks of life, but the palate is not usually repaired until the child is 18 months old."

4. "The lip is repaired in the first few weeks of life, but the palate is not usually repaired until the child is 18 months old."

The nurse will soon receive a 4-month-old who has been diagnosed with intussusception. The infant is described as very lethargic with the following vital signs: T 101.8°F (38.7°C), HR 181, BP 68/38. The reporting nurse states the infant's abdomen is very rigid. Which is the most appropriate action for the receiving nurse? 1. Prepare to accompany the infant to a computed tomography scan to confirm the diagnosis. 2. Prepare to accompany the infant to the radiology department for a reducing enema. 3. Prepare to start a second intravenous line to administer fluids and antibiotics. 4. Prepare to get the infant ready for immediate surgical correction.

4. Prepare to get the infant ready for immediate surgical correction. Intussusception with peritonitis is a surgical emergency, so preparing the infant for surgery is the nurse's top priority.

Which should be the nurse's immediate action when a newborn begins to cough and choke and becomes cyanotic while feeding? 1. Inform the physician of the situation. 2. Have the mother stop feeding the infant, and observe to see if the choking episode resolves on its own. 3. Immediately determine the infant's oxygen saturation, and have the mother stop feeding the infant. 4. Take the infant from the mother, and administer blow-by oxygen while obtaining the infant's oxygen saturation.

4. Take the infant from the mother, and administer blow-by oxygen while obtaining the infant's oxygen saturation.

18. An 8-month-old infant is brought to the clinic for evaluation. The mother tells the nurse that she has noticed some white patches on the infant's tongue that look like curdled milk after breastfeeding. The nurse suspects oral candidiasis (thrush). Which question would the nurse use to help confirm this suspicion? A) "Are you having breast pain when you nurse the baby?" B) "Has he had any dairy problems recently?" C) "Is he experiencing any vomiting lately?" D) "How have his stools been this past week?"

Ans: A Feedback: The infant may develop thrush from the mother if the mother has a fungal infection of the breast. Asking the mother about breast pain would be important because this type of infection can cause the mother a great deal of pain with nursing. Dairy products are not associated with oral candidiasis but are associated with the development of infectious diarrhea in infants. Vomiting is unrelated to thrush. The infant also may have candidal diaper rash, but this would be manifested on the skin as a beefy-red rash with satellite lesions, not in his stools.

22. The nurse is providing care to a child with pancreatitis. When reviewing the child's laboratory test results, what would the nurse expect to find? Select all that apply. A) Leukocytosis B) Decreased C-reactive protein C) Elevated serum amylase levels D) Positive stool culture E) Decreased serum lipase levels

Ans: A, C Feedback: With pancreatitis, serum amylase and lipase levels are elevated and levels three times the normal values are extremely indicative of pancreatitis. Leukocytosis is common with acute pancreatitis. C-reactive protein levels may be elevated. Stool cultures are not used to evaluate this disorder. Positive stool cultures would indicate a bacterial cause of diarrhea.

21. The parents of a boy diagnosed with Hirschsprung disease are anxious and fearful of the upcoming surgery. The mother states, "I'm worried about having to care for our son's ostomy." Which intervention would be most helpful for the parents? A) Explaining to them about the diagnosis and surgery B) Having a wound, ostomy, and continence nurse meet with them C) Reinforcing that the ostomy will be temporary D) Teaching them about the medications used to slow stool output

Ans: B Feedback: Although explaining about the diagnosis and surgery, reinforcing that the ostomy will be temporary, and teaching them about medications would be appropriate, the parents are voicing concerns about caring for the ostomy. Therefore, having a wound, ostomy, and continence nurse meet with them would address these concerns and help them deal with the anxieties and care of a newly placed stoma.

16. The mother of a 3-week-old infant old brings her daughter in for an evaluation. During the visit, the mother tells the nurse that her baby is spitting up after feedings. Which response by the nurse would be most appropriate? A) "We need to tell the healthcare provider about this." B) "Infants this age commonly spit up." C) "Your daughter might have an allergy." D) "Don't worry; you're just feeding her too much."

Ans: B Feedback: In infants younger than 1 month of age, the lower esophageal sphincter is not fully developed, so infants younger than 1 month of age frequently regurgitate after feedings. Many children younger than 1 year of age continue to regurgitate for several months, but this usually disappears with age. The mother's report is not a cause for concern, so the healthcare provider does not need to be notified. Additional information would be needed to determine if the infant had an allergy. Although the infant's stomach capacity is small, telling the mother not to worry does not address the mother's concern, and telling her that she is feeding the daughter too much implies that she is doing something wrong.

10. The school nurse is working with a 10-year-old girl with recurrent abdominal pain. The girl's teacher has been less than understanding about the frequent absences and trips to the nurse's office. How should the nurse respond? A) "Be patient; she is trying some new medication." B) "The pain she is having is real." C) "The family is working toward improvement." D) "Please do not add to this family's stress."

Ans: B Feedback: It is important to educate the teacher that this recurrent abdominal pain is a true pain that the child feels and it is not "in her mind." Telling the teacher not to add to the family's stress or that the family is working toward improvement does not teach. The nurse must have the permission of the family to discuss the girl's medication.

9. The nurse is caring for a 2-month-old with a cleft palate. The child will undergo corrective surgery at age 3 months. The mother would like to continue breastfeeding the baby after surgery and wonders if it is possible. How should the nurse respond? A) "There is a good chance that you will be able to breastfeed almost immediately." B) "Breastfeeding is likely to be possible but check with the surgeon." C) "After the suture line heals, breastfeeding can resume." D) "We will have to wait and see what happens after the surgery."

Ans: B Feedback: Postoperatively, some surgeons allow breastfeeding to be resumed almost immediately. However, the nurse needs to advise the mother to check with the surgeon to determine when breastfeeding can resume. Telling the mother that she has to wait until the suture line heals may be inaccurate. Telling her to wait and see does not answer her question.

13. The nurse is determining maintenance fluid requirements for a child who weighs 25 kg. How much fluid would the child need per day? A) 1,560 mL B) 1,600 mL C) 1,650 mL D) 1,700 mL

Ans: B Feedback: Using the following formula of 100 mL/kg for the first 10 kg, plus 50 mL/kg for the next 10 kg, and then 20 mL/kg for the remaining kg, the child would require (100 × 10) + (50 × 10) + (20 × 5) = 1,000 + 500 + 100 = 1,600 mL in 24 hours.

19. The parents of a 6-week-old boy come to the clinic for evaluation because the infant has been vomiting. The parents report that the vomiting has been increasing in frequency and forcefulness over the last week. The mother says, "Sometimes, it seems like it just bursts out of his mouth." A diagnosis of hypertrophic pyloric stenosis is suspected. When performing the physical examination, what would the nurse most likely find? A) Sausage-shaped mass in the upper midabdomen B) Hard, moveable, olive-shaped mass in the right upper quadrant C) Tenderness over the McBurney point in the right lower quadrant D) Abdominal pain in the epigastric or umbilical region

Ans: B Feedback: With hypertrophic pyloric stenosis, a hard, moveable, olive-shaped mass would be palpated in the right upper quadrant. A sausage-shaped mass in the upper midabdomen would suggest intussusception. Tenderness over the McBurney point would be associated with appendicitis. Epigastric or umbilical pain would be associated with peptic ulcer disease.

28. The nurse is caring for a 6-month-old with a cleft lip and palate. The mother of the child demonstrates understanding of the disorder with which statements? Select all that apply. A) "My smoking during pregnancy didn't have anything to do with this disorder. Smoking primarily causes low birth weight." B) "I know my baby takes a lot longer to feed than most children this age." C) "It really worries me that my baby may have some other disorders that haven't been detected yet." D) "I wonder if my baby will develop speech problems when language development begins?" E) "Thankfully there are healthcare providers that specialize in correcting this type of disorder."

Ans: B, C, D, E Feedback: Feeding and speech are especially difficult for the child with cleft lip and palate until the defect is repaired. Cleft lip and palate occurs frequently in association with other anomalies and has been identified in more than 350 syndromes. Plastic surgeons or craniofacial specialists, oral surgeons, dentists or orthodontists, and prosthodontists are some of the healthcare providers that specialize in repair of this disorder. The mother is incorrect in stating that smoking is not associated with cleft lip or palate. Maternal smoking during pregnancy is a major risk factor for the disorder.

25. After teaching the parents of a child diagnosed with celiac disease about nutrition, the nurse determines that the teaching was effective when the parents identify which foods as appropriate for their child? Select all that apply. A) Wheat germ B) Peanut butter C) Carbonated drinks D) Shellfish E) Jelly F) Flavored yogurt

Ans: B, C, D, E Feedback: Foods allowed in a gluten-free diet include peanut butter, carbonated drinks, shellfish, and jelly. Wheat germ and flavored yogurt should be avoided.

29. The nurse is performing a gastrointestinal assessment on a 7-year-old boy. The parents are assisting with the history. Which assessment findings are indicative of constipation? Select all that apply. A) "Our child only has 3 to 4 bowel movements per week." B) "Our child complains of pain because his bowel movements are so hard." C) "Our child tells us that his belly hurts a lot of the time." D) "I can tell he holds his bowel movement much of the time because of the way he stands." E) "I find smears of stool in his underwear almost every day."

Ans: B, C, D, E Feedback: Pain, stool withholding behavior (retentive posturing), and encopresis (soiling of fecal contents into the underwear beyond the age of expected toilet training) are all signs of chronic functional constipation. Less than 3 bowel movements is considered constipation.

11. When examining the abdomen of a child, which technique would the nurse use last? A) Auscultation B) Percussion C) Palpation D) Inspection

Ans: C Feedback: Palpation should be the last part of the abdominal examination. Inspection, auscultation, and percussion should be done before palpation.

23. A child is scheduled for a lower endoscopy. What would the nurse include in the child's plan of care in preparation for this test? A) Explaining about the need to ingest barium B) Establishing an intravenous access for radionuclide administration C) Administering the prescribed bowel cleansing regimen D) Withholding prescribed proton pump inhibitors for 5 days before

Ans: C Feedback: Prior to a lower endoscopy, the child must undergo bowel cleansing to allow visualization of the lower gastrointestinal tract via a fiberoptic instrument. Barium is ingested for an upper gastrointestinal and/or small bowel series. Radionuclides are used with a hepatobiliary scan. Proton pump inhibitors are withheld for 5 days before a urea breath test.

15. The nurse is providing care to a child with an intussusception. The child has a bowel movement and the nurse inspects the stool. The nurse would most likely document the stool's appearance as having what quality? A) Greasy B) Clay-colored C) Currant jelly-like D) Bloody

Ans: C Feedback: The child with intussusception often exhibits currant jelly-like stools that may or may not be positive for blood. Greasy stools are associated with celiac disease. Cay-colored stools are observed with biliary atresia. Bloody stools can be seen with several gastrointestinal disorders, such as inflammatory bowel disease.

14. The parents of a child diagnosed with celiac disease ask the nurse what types of food they can offer their child. What recommendation would the nurse include in the teaching plan? A) Frozen yogurt B) Rye bread C) Creamed spinach D) Fruit juice

Ans: D Feedback: For the child with celiac disease, foods containing gluten such as frozen yogurt, rye bread, and creamed vegetables should be avoided. Fruit juice would be an appropriate suggestion in a gluten-free diet.

8. The nurse has developed a plan of care for a 12-month-old hospitalized with dehydration as a result of rotavirus. Which intervention would the nurse include in the plan of care? A) Encouraging consumption of fruit juice B) Offering Kool-Aid or popsicles as tolerated C) Encouraging milk products to boost caloric intake D) Maintaining the intravenous (IV) fluid rate as ordered

Ans: D Feedback: The nurse should maintain an IV line and administer the IV fluid as ordered to maintain fluid volume. High-carbohydrate fluids like fruit juice, Kool-Aid, and popsicles should be avoided as they are low in electrolytes, increase simple carbohydrate consumption, and can decrease stool transit time. Milk products should be avoided during the acute phase of illness as they may worsen diarrhea.

17. A group of students are reviewing information about fluid balance and losses in children in comparison to adults. The students demonstrate a need for additional review when they state that: A) children have a proportionately greater amount of body water than do adults. B) fever plays a greater role in insensible fluid losses in infants and children. C) a higher metabolic rate plays a major role in increased insensible fluid losses. D) the infant's immature kidneys have a tendency to overconcentrate urine.

Ans: D Feedback: The young infant's renal immaturity does not allow the kidneys to concentrate urine as well as in older children and adults, placing them at risk for dehydration or overhydration. Children do have a proportionately greater amount of body water than adults, and fever is important in promoting insensible fluid losses in infants and children because children become febrile more readily and their fevers are higher than those in adults. Children also experience a higher metabolic rate, which accounts for increased insensible fluid losses and increased need for water for excretory function.


Conjuntos de estudio relacionados

AU 60 - Assignment 6 - Analyzing Financial Statements

View Set

C1:M1: Why Cloud Tech is Revolutionizing Business

View Set

Chapter 3 How to work with the primitive data type Murach's Java Programming

View Set

Peripheral Vascular Disease/Peripheral Arterial Disease Prep-U/Test-Bank

View Set

Micro - Economics : EXAM 3 MULTIPLE CHOICE QUESTIONS

View Set